R N352 PrepU Chapter 51 - Diabetes

Pataasin ang iyong marka sa homework at exams ngayon gamit ang Quizwiz!

Which clinical characteristic is associated with type 1 diabetes (previously referred to as insulin-dependent diabetes mellitus)? a) Requirement for oral hypoglycemic agents b) Presence of islet cell antibodies c) Obesity d) Rare ketosis

Presence of islet cell antibodies Explanation: Individuals with type 1 diabetes often have islet cell antibodies and are usually thin or demonstrate recent weight loss at the time of diagnosis. These individuals are prone to experiencing ketosis when insulin is absent and require exogenous insulin to preserve life.

A client who was diagnosed with type 1 diabetes 14 years ago is admitted to the medical-surgical unit with abdominal pain. On admission, the client's blood glucose level is 470 mg/dl. Which finding is most likely to accompany this blood glucose level? A. Arm and leg trembling B. Rapid, thready pulse C. Slow, shallow respirations D. Cool, moist skin

Rapid, thready pulse This client's abnormally high blood glucose level indicates hyperglycemia, which typically causes polyuria, polyphagia, and polydipsia. Because polyuria leads to fluid loss, the nurse should expect to assess signs of deficient fluid volume, such as a rapid, thready pulse; decreased blood pressure; and rapid respirations. Cool, moist skin and arm and leg trembling are associated with hypoglycemia. Rapid respirations — not slow, shallow ones — are associated with hyperglycemia.

Lispro (Humalog) is an example of which type of insulin? a) Short-acting b) Intermediate-acting c) Rapid-acting d) Long-acting

Rapid-acting Humalog is a rapid-acting insulin. NPH is an intermediate-acting insulin. A short-acting insulin is Humulin-R. An example of a long-acting insulin is Glargine (Lantus).

What is the only insulin that can be given intravenously? A. Regular B. Ultralente C. Lantus D. NPH

Regular Insulins other than regular are in suspensions that could be harmful if administered IV.

A client is admitted to the unit with diabetic ketoacidosis (DKA). Which insulin would the nurse expect to administer intravenously? a) Glargine b) Lente c) Regular d) NPH

Regular Regular insulin is administered intravenously to treat DKA. It is added to an IV solution and infused continuously. Glargine, NPH, and Lente are only administered subcutaneously.

The nurse is administering an insulin drip to a patient in ketoacidosis. What insulin does the nurse know is the only one that can be used intravenously? A. Regular B. Lantus C. Lispro D. NPH

Regular Short-acting insulins are called regular insulin (marked R on the bottle). Regular insulin is a clear solution and is usually administered 20 to 30 minutes before a meal, either alone or in combination with a longer-acting insulin. Regular insulin is the only insulin approved for IV use.

A patient has been newly diagnosed with type 2 diabetes, and the nurse is assisting with the development of a meal plan. What step should be taken into consideration prior to making the meal plan? a) Determining whether the patient is on insulin or taking oral antidiabetic medication b) Ensuring that the patient understands that some favorite foods may not be allowed on the meal plan and substitutes will need to be found c) Reviewing the patient's diet history to identify eating habits and lifestyle and cultural eating patterns d) Making sure that the patient is aware that quantity of foods will be limited

Reviewing the patient's diet history to identify eating habits and lifestyle and cultural eating patterns The first step in preparing a meal plan is a thorough review of the patient's diet history to identify eating habits and lifestyle and cultural eating patterns.

The diabetic client asks the nurse why shoes and socks are removed at each office visit. Which assessment finding is most significant in determining the protocol for inspection of feet? A. Sensory neuropathy B. Retinopathy C. Autonomic neuropathy D. Nephropathy

Sensory neuropathy Neuropathy results from poor glucose control and decreased circulation to nerve tissues. Neuropathy involving sensory nerves located in the periphery can lead to lack of sensitivity, which increases the potential for soft tissue injury without client awareness. The feet are inspected on each visit to insure no injury or pressure has occurred. Autonomic neuropathy, retinopathy, and nephropathy affect nerves to organs other than feet.

A client with diabetes mellitus has a prescription for 5 units of U-100 regular insulin and 25 units of U-100 isophane insulin suspension (NPH) to be taken before breakfast. At about 4:30 p.m., the client experiences headache, sweating, tremor, pallor, and nervousness. What is the most probable cause of these signs and symptoms? A. Serum glucose level of 52 mg/dl B. Serum calcium level of 10.2 mg/dl C. Serum glucose level of 450 mg/dl D. Serum calcium level of 8.9 mg/dl

Serum glucose level of 52 mg/dL Headache, sweating, tremor, pallor, and nervousness typically result from hypoglycemia, an insulin reaction in which serum glucose level drops below 70 mg/dl. Hypoglycemia may occur 4 to 18 hours after administration of isophane insulin suspension or insulin zinc suspension (Lente), which are intermediate-acting insulins. Although hypoglycemia may occur at any time, it usually precedes meals. Hyperglycemia, in which serum glucose level is above 180 mg/dl, causes such early manifestations as fatigue, malaise, drowsiness, polyuria, and polydipsia. A serum calcium level of 8.9 mg/dl or 10.2 mg/dl is within normal range and wouldn't cause the client's symptoms.

A client with status asthmaticus requires endotracheal intubation and mechanical ventilation. Twenty-four hours after intubation, the client is started on the insulin infusion protocol. The nurse must monitor the client's blood glucose levels hourly and watch for which early signs and symptoms associated with hypoglycemia? a) Sweating, tremors, and tachycardia b) Dry skin, bradycardia, and somnolence c) Polyuria, polydipsia, and polyphagia d) Bradycardia, thirst, and anxiety

Sweating, tremors, and tachycardia Sweating, tremors, and tachycardia, thirst, and anxiety are early signs of hypoglycemia. Dry skin, bradycardia, and somnolence are signs and symptoms associated with hypothyroidism. Polyuria, polydipsia, and polyphagia are signs and symptoms of diabetes mellitus.

Which of the following factors would a nurse identify as a most likely cause of diabetic ketoacidosis (DKA) in a client with diabetes? a) The client has eaten and has not taken or received insulin. b) The client has not consumed sufficient calories. c) The client continues medication therapy despite adequate food intake. d) The client has been exercising more than usual.

The client has eaten and has not taken or received insulin. If the client has eaten and has not taken or received insulin, DKA is more likely to develop. Hypoglycemia is more likely to develop if the client has not consumed food and continues to take insulin or oral antidiabetic medications, if the client has not consumed sufficient calories, or if client has been exercising more than usual.

Which may be a potential cause of hypoglycemia in the client diagnosed with diabetes mellitus? A. The client has eaten but has not taken or received insulin. B. The client has not eaten but continues to take insulin or oral antidiabetic medications. C. The client has not been exercising. D. The client has not complied with the prescribed treatment regimen.

The client has not eaten but continues to take insulin or oral antidiabetic medications. Hypoglycemia occurs when a client with diabetes is not eating and continues to take insulin or oral antidiabetic medications. Hypoglycemia does not occur when the client has not been compliant with the prescribed treatment regimen. If the client has eaten and has not taken or received insulin, diabetic ketoacidosis is more likely to develop.

A male client, aged 42, is diagnosed with diabetes mellitus. He visits the gym regularly and is a vegetarian. Which of the following factors is important when assessing the client? A. History of radiographic contrast studies that used iodine B. The client's exercise routine C. The client's mental and emotional status D. The client's consumption of carbohydrates

The client's consumption of carbohydrates While assessing a client, it is important to note the client's consumption of carbohydrates because he has high blood sugar. Although other factors such as the client's mental and emotional status, history of tests involving iodine, and exercise routine can be part of data collection, they are not as important to information related to the client's to be noted in a client with high blood sugar.

A client with diabetes mellitus develops sinusitis and otitis media accompanied by a temperature of 100.8° F (38.2° C). What effect do these findings have on his need for insulin? a) They decrease the need for insulin. b) They increase the need for insulin. c) They cause wide fluctuations in the need for insulin. d) They have no effect.

They increase the need for insulin. Insulin requirements increase in response to growth, pregnancy, increased food intake, stress, surgery, infection, illness, increased insulin antibodies, and some medications. Insulin requirements are decreased by hypothyroidism, decreased food intake, exercise, and some medications.

When the nurse is caring for a patient with type 1 diabetes, what clinical manifestation would be a priority to closely monitor? -Hypoglycemia -Hyponatremia -Ketonuria -Polyphagia

-Hypoglycemia The therapeutic goal for diabetes management is to achieve normal blood glucose levels (euglycemia) without hypoglycemia while maintaining a high quality of life.

Which instruction about insulin administration should a nurse give to a client? A. "Shake the vials before withdrawing the insulin." B. "Always follow the same order when drawing the different insulins into the syringe." C. "Discard the intermediate-acting insulin if it appears cloudy." D. "Store unopened vials of insulin in the freezer at temperatures well below freezing."

"Always follow the same order when drawing different insulins into the syringe." The nurse should instruct the client to always follow the same order when drawing the different insulins into the syringe. Insulin should never be shaken because the resulting froth prevents withdrawal of an accurate dose and may damage the insulin protein molecules. Insulin should never be frozen because the insulin protein molecules may be damaged. The client doesn't need to discard intermediate-acting insulin if it's cloudy; this finding is normal.

The nurse is providing information about foot care to a client with diabetes. Which of the following would the nurse include? A. "Be sure to apply a moisturizer to feet daily." B. "Wash your feet in hot water every day." C. "Use a razor to remove corns or calluses." D. "Wear well-fitting comfortable rubber shoes."

"Be sure to apply a moisturizer to feet daily." The nurse should advise the client to apply a moisturizer to the feet daily. The client should use warm water not hot water to bathe his feet. Razors to remove corns or calluses must be avoided to prevent injury and infection. The client should wear well-fitting comfortable shoes, avoiding shoes made of rubber, plastic or vinyl which would cause the feet to perspire.

The nurse is educating a patient about the benefits of fruit versus fruit juice in the diabetic diet. The patient states, "What difference does it make if you drink the juice or eat the fruit? It is all the same." What is the best response by the nurse? A. "Eating the fruit is more satisfying than drinking the juice. You will get full faster." B. "Eating the fruit instead of drinking juice decreases the glycemic index by slowing absorption." C. "Eating the fruit will give you more vitamins and minerals than the juice will." D. "The fruit has less sugar than the juice."

"Eating the fruit instead of drinking juice decreases the glycemic index by slowing absorption." Eating whole fruit instead of drinking juice decreases the glycemic index, because fiber in the fruit slows absorption.

After teaching a client with type 1 diabetes who is scheduled to undergo an islet cell transplant, which client statement indicates successful teaching? a. "This transplant will provide me with a cure for my diabetes." b. "I will receive a whole organ with extra cells to produce insulin." c. "They'll need to create a connection from the pancreas to allow enzymes to drain." d. "I might need insulin later on but probably not as much or as often."

"I might need insulin later on but probably not as much or as often." Transplanted islet cells tend to lose their ability to function over time, and approximately 70% of recipients resume insulin administration in 2 years. However, the amount of insulin and the frequency of its administration are reduced because of improved control of blood glucose levels. Thus, this type of transplant doesn't cure diabetes. It requires the use of two human pancreases to obtain sufficient numbers of islet cells for transplantation. A whole organ transplant requires a means for exocrine enzyme drainage and venous absorption of insulin.

A client is evaluated for type 1 diabetes. Which client comment correlates best with this disorder? A. "I'm thirsty all the time. I just can't get enough to drink." B. "I notice pain when I urinate." C. "It seems like I have no appetite. I have to make myself eat." D. "I have a cough and cold that just won't go away."

"I'm thirsty all the time. I just can't get enough to drink." The nurse should instruct a client with diabetes mellitus to check his blood glucose levels every 3 to 4 hours and take insulin or an oral antidiabetic agent as usual, even when he's sick. If the client's blood glucose level rises above 300 mg/dl, he should call his physician immediately. If the client is unable to follow the regular meal plan because of nausea, he should substitute soft foods, such as gelatin, soup, and custard.

A nurse is teaching a client recovering from diabetic ketoacidosis (DKA) about management of "sick days." The client asks the nurse why it is important to monitor the urine for ketones. Which statement is the nurse's best response? a) "Ketones are formed when insufficient insulin leads to cellular starvation. As cells rupture, they release these acids into the blood." b) "Excess glucose in the blood is metabolized by the liver and turned into ketones, which are an acid." c) "Ketones accumulate in the blood and urine when fat breaks down in the absence of insulin. Ketones signal an insulin deficiency that will cause the body to start breaking down stored fat for energy." d) "When the body does not have enough insulin, hyperglycemia occurs. Excess glucose is broken down by the liver, causing acidic by-products to be released."

"Ketones accumulate in the blood and urine when fat breaks down in the absence of insulin. Ketones signal an insulin deficiency that will cause the body to start breaking down stored fat for energy." Ketones (or ketone bodies) are by-products of fat breakdown in the absence of insulin, and they accumulate in the blood and urine. Ketones in the urine signal an insulin deficiency and that control of type 1 diabetes is deteriorating. When almost no effective insulin is available, the body starts to break down stored fat for energy.

A client with type 1 diabetes mellitus is being taught about self-injection of insulin. Which fact about site rotation should the nurse include in the teaching? -Avoid the abdomen because absorption there is irregular. -Choose a different site at random for each injection. -Rotate sites from area to area every other day. -Use all available injection sites within one area.

-Use all available injection sites within one area. Systematic rotation of injection sites within an anatomic area is recommended to prevent localized changes in fatty tissue. To promote consistency in insulin absorption, the client should be encouraged to use all available injection sites within one area rather than randomly rotating sites from area to area.

A client newly diagnosed with diabetes mellitus asks why he needs ketone testing when the disease affects his blood glucose levels. How should the nurse respond? A. "Ketones can damage your kidneys and eyes." B. "Ketones will tell us if your body is using other tissues for energy." C. "Ketones help the physician determine how serious your diabetes is." D. "The spleen releases ketones when your body can't use glucose."

"Ketones will tell us if your body is using other tissues for energy." The nurse should tell the client that ketones are a byproduct of fat metabolism and that ketone testing can determine whether the body is breaking down fat to use for energy. The spleen doesn't release ketones when the body can't use glucose. Although ketones can damage the eyes and kidneys and help the physician evaluate the severity of a client's diabetes, these responses by the nurse are incomplete.

A client with diabetes mellitus must learn how to self-administer insulin. The physician has ordered 10 units of U-100 regular insulin and 35 units of U-100 isophane insulin suspension (NPH) to be taken before breakfast. When teaching the client how to select and rotate insulin injection sites, the nurse should provide which instruction? A. "Rotate injection sites within the same anatomic region, not among different regions." B. "Administer insulin into sites above muscles that you plan to exercise heavily later that day." C. "Inject insulin into healthy tissue with large blood vessels and nerves." D. "Administer insulin into areas of scar tissue or hypertrophy whenever possible."

"Rotate injection sites within the same anatomic region, not among different regions." The nurse should instruct the client to rotate injection sites within the same anatomic region. Rotating sites among different regions may cause excessive day-to-day variations in the blood glucose level; also, insulin absorption differs from one region to the next. Insulin should be injected only into healthy tissue lacking large blood vessels, nerves, or scar tissue or other deviations. Injecting insulin into areas of hypertrophy may delay absorption. The client shouldn't inject insulin into areas of lipodystrophy (such as hypertrophy or atrophy); to prevent lipodystrophy, the client should rotate injection sites systematically. Exercise speeds drug absorption, so the client shouldn't inject insulin into sites above muscles that will be exercised heavily.

Which instruction should a nurse give to a client with diabetes mellitus when teaching about "sick day rules"? a) "Don't take your insulin or oral antidiabetic agent if you don't eat." b) "Test your blood glucose every 4 hours." c) "Follow your regular meal plan, even if you're nauseous." d) "It's okay for your blood glucose to go above 300 mg/dl while you're sick."

"Test your blood glucose every 4 hours." The nurse should instruct a client with diabetes mellitus to check his blood glucose levels every 3 to 4 hours and take insulin or an oral antidiabetic agent as usual, even when he's sick. If the client's blood glucose level rises above 300 mg/dl, he should call his physician immediately. If the client is unable to follow the regular meal plan because of nausea, he should substitute soft foods, such as gelatin, soup, and custard.

Health teaching for a patient with diabetes who is prescribed Humulin N, an intermediate NPH insulin, would include which of the following advice? a) "Your insulin will begin to act in 15 minutes." b) "Your insulin will last 8 hours, and you will need to take it three times a day." c) "You should take your insulin after you eat breakfast and dinner." d) "You should expect your insulin to reach its peak effectiveness by 12 noon if you take it at 8:00 AM."

"You should take your insulin after you eat breakfast and dinner."

A client has just been diagnosed with type 1 diabetes. When teaching the client and family how diet and exercise affect insulin requirements, the nurse should include which guideline? A. "You'll need more insulin when you exercise or decrease your food intake." B. "You'll need less insulin when you exercise or reduce your food intake." C. "You'll need more insulin when you exercise or increase your food intake." D. "You'll need less insulin when you increase your food intake."

"You'll need less insulin when you exercise or reduce your food intake." The nurse should advise the client that exercise, reduced food intake, hypothyroidism, and certain medications decrease insulin requirements. Growth, pregnancy, greater food intake, stress, surgery, infection, illness, increased insulin antibodies, and certain medications increase insulin requirements.

A 16-year-old client newly diagnosed with type 1 diabetes has a very low body weight despite eating regular meals. The client is upset because friends frequently state, "You look anorexic." Which statement by the nurse would be the best response to help this client understand the cause of weight loss due to this condition? a) "I will refer you to a dietician who can help you with your weight." b) "You may be having undiagnosed infections, causing you to lose extra weight." c) "Don't worry about what your friends think; the carbohydrates you eat are being quickly digested, increasing your metabolism." d) "Your body is using protein and fat for energy instead of glucose."

"Your body is using protein and fat for energy instead of glucose." Persons with type 1 diabetes, particularly those in poor control of the condition, tend to be thin because when the body cannot effectively utilize glucose for energy (no insulin supply), it begins to break down protein and fat as an alternate energy source. Patients may be underweight at the onset of type 1 diabetes because of rapid weight loss from severe hyperglycemia. The goal initially may be to provide a higher-calorie diet to regain lost weight and blood glucose control.

A nurse is preparing to administer two types of insulin to a client with diabetes mellitus. What is the correct procedure for preparing this medication? -The short-acting insulin is withdrawn before the intermediate-acting insulin. -The intermediate-acting insulin is withdrawn before the short-acting insulin. -Different types of insulin are not to be mixed in the same syringe. -If administered immediately, there is no requirement for withdrawing one type of insulin before another.

-The short-acting insulin is withdrawn before the intermediate-acting insulin. When combining two types of insulin in the same syringe, the short-acting regular insulin is withdrawn into the syringe first and the intermediate-acting insulin is added next. This practice is referred to as "clear to cloudy."

A physician orders blood glucose levels every 4 hours for a 4-year-old child with brittle type 1 diabetes. The parents are worried that drawing so much blood will traumatize their child. How can the nurse best reassure the parents? A. "Your child will need less blood work as his glucose levels stabilize." B. "Our laboratory technicians use tiny needles and they're really good with children." C. "Your child is young and will soon forget this experience." D. "I'll see if the physician can reduce the number of blood draws."

"Your child will need less blood work as his glucose levels stabilize." Telling the parents that the number of blood draws will decrease as their child's glucose levels stabilize engages them in the learning process and gives them hope that the present discomfort will end as the child's condition improves. Telling the parents that their child won't remember the experience disregards their concerns and anxiety. The nurse shouldn't offer to ask the physician if he can reduce the number of blood draws; the physician needs the laboratory results to monitor the child's condition properly. Although telling the parents that the laboratory technicians are gentle and use tiny needles may be reassuring, it isn't the most appropriate response.

A client with diabetes mellitus has a blood glucose level of 40 mg/dL. Which rapidly absorbed carbohydrate would be most effective? -1/2 cup fruit juice or regular soft drink -4 oz of skim milk -1/2 tbsp honey or syrup -three to six LifeSavers candies

-1/2 cup fruit juice or regular soft drink In a client with hypoglycemia, the nurse uses the rule of 15: give 15 g of rapidly absorbed carbohydrate, wait 15 minutes, recheck the blood sugar, and administer another 15 g of glucose if the blood sugar is not above 70 mg/dL. One-half cup fruit juice or regular soft drink is equivalent to the recommended 15 g of rapidly absorbed carbohydrate. Eight ounces of skim milk is equivalent to the recommended 15 g of rapidly absorbed carbohydrate. One tablespoon of honey or syrup is equivalent to the recommended 15 g of rapidly absorbed carbohydrate. Six to eight LifeSavers candies is equivalent to the recommended 15 g of rapidly absorbed carbohydrate.

Once digested, what percentage of carbohydrates is converted to glucose? -70 -80 -90 -100

-100 Once digested, 100% of carbohydrates are converted to glucose. However, approximately 50% of protein foods are also converted to glucose, but this has minimal effect on blood glucose concentration.

blood glucose level of 800 mg/dl. Which solution is the most appropriate at the beginning of therapy? -100 units of regular insulin in normal saline solution -100 units of neutral protamine Hagedorn (NPH) insulin in normal saline solution -100 units of regular insulin in dextrose 5% in water -100 units of NPH insulin in dextrose 5% in water

-100 units of regular insulin in normal saline solution Continuous insulin infusions use only short-acting regular insulin. Insulin is added to normal saline solution and administered until the client's blood glucose level falls. Further along in the therapy, a dextrose solution is administered to prevent hypoglycemia.

A child is brought into the emergency department with vomiting, drowsiness, and blowing respirations. The father reports that the symptoms have been progressing throughout the day. The nurse suspects diabetic ketoacidosis (DKA). Which action should the nurse take first in the management of DKA? -Give prescribed antiemetics. -Begin fluid replacements. -Administer prescribed dose of insulin. -Administer bicarbonate to correct acidosis.

-Begin fluid replacements. Management of DKA is aimed at correcting dehydration, electrolyte loss, and acidosis before correcting the hypoglycemia with insulin.

A nurse expects to note an elevated serum glucose level in a client with hyperosmolar hyperglycemic nonketotic syndrome (HHNS). Which other laboratory finding should the nurse anticipate? -Elevated serum acetone level -Serum ketone bodies -Serum alkalosis -Below-normal serum potassium level

-Below-normal serum potassium level A client with HHNS has an overall body deficit of potassium resulting from diuresis, which occurs secondary to the hyperosmolar, hyperglycemic state caused by the relative insulin deficiency. An elevated serum acetone level and serum ketone bodies are characteristic of diabetic ketoacidosis. Metabolic acidosis, not serum alkalosis, may occur in HHNS.

Which of the following is a characteristic of diabetic ketoacidosis (DKA)? Select all that apply. -Elevated blood urea nitrogen (BUN) and creatinine -Rapid onset -More common in type 1 diabetes -Absent ketones -Normal arterial pH level

-Elevated blood urea nitrogen (BUN) and creatinine -Rapid onset -More common in type 1 diabetes DKA is characterized by an elevated BUN and creatinine, rapid onset, and it is more common in type 1 diabetes. Hyperglycemic hyperosmolar nonketotic syndrome (HHNS) is characterized by the absence of urine and serum ketones and a normal arterial pH level.

Which assessment finding is most important in determining nursing care for a client with diabetes mellitus? -Respirations of 12 breaths/minute -Cloudy urine -Blood sugar 170 mg/dL -Fruity breath

-Fruity breath The rising ketones and acetone in the blood can lead to acidosis and be detected as a fruity odor on the breath. Ketoacidosis needs to be treated to prevent further complications such as Kussmaul respirations (fast, labored breathing) and renal shutdown. A blood sugar of 170 mg/dL is not ideal but will not result in glycosuria and/or trigger the classic symptoms of diabetes mellitus. Cloudy urine may indicate a UTI.

A nurse is preparing to discharge a client with coronary artery disease and hypertension who is at risk for type 2 diabetes. Which information is important to include in the discharge teaching? -How to control blood glucose through lifestyle modification with diet and exercise -How to self-inject insulin -How to monitor ketones daily -How to recognize signs of diabetic ketoacidosis

-How to control blood glucose through lifestyle modification with diet and exercise Persons at high risk for type 2 diabetes receive standard lifestyle recommendations plus metformin, standard lifestyle recommendations plus placebo, or an intensive program of lifestyle modifications. The 16-lesson curriculum of the intensive program of lifestyle modifications focuses on reducing weight by more than 7% of initial body weight and moderate-intensity physical activity. It also includes behavior modification strategies designed to help clients achieve the goals of weight reduction and participation in exercise. These findings demonstrate that type 2 diabetes can be prevented or delayed in persons at high risk for the disease.

Which of the following is a risk factor for the development of diabetes mellitus? Select all that apply. -Hypertension -Obesity -Family history -Age greater of 45 years or older -History of gestational diabetes

-Hypertension -Obesity -Family history -Age greater of 45 years or older -History of gestational diabetes

The nurse is educating the client with diabetes on setting up a sick plan to manage blood glucose control during times of minor illness such as influenza. Which is the most important teaching item to include? -Increase frequency of glucose self-monitoring. -Decrease food intake until nausea passes. -Do not take insulin if not eating. -Take half the usual dose of insulin until symptoms resolve.

-Increase frequency of glucose self-monitoring. Minor illnesses such as influenza can present a special challenge to a diabetic client. The body's need for insulin increases during illness. Therefore, the client should take the prescribed insulin dose, increase the frequency of glucose monitoring, and maintain adequate fluid intake to counteract the dehydrating effects of hyperglycemia. Clear liquids and juices are encouraged. Taking less than normal dose of insulin may lead to ketoacidosis.

The nurse is describing the action of insulin in the body to a client newly diagnosed with type 1 diabetes. Which of the following would the nurse explain as being the primary action? -It enhances transport of glucose across the cell wall. -It aids in the process of gluconeogenesis. -It stimulates the pancreatic beta cells. -It decreases the intestinal absorption of glucose.

-It enhances transport of glucose across the cell wall. Insulin carries glucose into body cells as their preferred source of energy. Besides, it promotes the liver's storage of glucose as glycogen and inhibits the breakdown of glycogen back into glucose. Insulin does not aid in gluconeogenesis but inhibits the breakdown of glycogen back into glucose.

The nurse is assessing a patient with nonproliferative (background) retinopathy. When examining the retina, what would the nurse expect to assess? Select all that apply. -Leakage of fluid or serum (exudates) -Microaneurysms -Focal capillary single closure -Detachment -Blurred optic discs

-Leakage of fluid or serum (exudates) -Microaneurysms -Focal capillary single closure Almost all patients with type 1 diabetes and the majority of patients with type 2 diabetes have some degree of retinopathy after 20 years (ADA, 2013). Changes in the microvasculature include microaneurysms, intraretinal hemorrhage, hard exudates, and focal capillary closure.

The nurse is caring for a client with an abnormally low blood glucose concentration. What glucose level will the nurse observe when assessing laboratory results? -Lower than 50 to 60 mg/dL (2.77 to 3.33 mmol/L) -Between 60 and 75 mg/dL (3.33 to 4.16 mmol/L) -Between 75 and 90 mg/dL (4.16 to 5.00 mmol/L) -95 mg/dL (5.27 mmol/L)

-Lower than 50 to 60 mg/dL (2.77 to 3.33 mmol/L) Hypoglycemia (low blood glucose) occurs when the blood glucose falls to less than 50 to 60 mg/dL (2.77 to 3.33 mmol/L).

A nurse is teaching a diabetic support group about the causes of type 1 diabetes. The teaching is determined to be effective when the group is able to attribute which factor as a cause of type 1 diabetes? -Presence of autoantibodies against islet cells -Obesity -Rare ketosis -Altered glucose metabolism

-Presence of autoantibodies against islet cells There is evidence of an autoimmune response in type 1 diabetes. This is an abnormal response in which antibodies are directed against normal tissues of the body, responding to these tissues as if they were foreign. Autoantibodies against islet cells and against endogenous (internal) insulin have been detected in people at the time of diagnosis and even several years before the development of clinical signs of type 1 diabetes.

Laboratory studies indicate a client's blood glucose level is 185 mg/dl. Two hours have passed since the client ate breakfast. Which test would yield the most conclusive diagnostic information about the client's glucose use? -Fasting blood glucose test -6-hour glucose tolerance test -Serum glycosylated hemoglobin (Hb A1c) -Urine ketones

-Serum glycosylated hemoglobin (Hb A1c) Hb A1c is the most reliable indicator of glucose use because it reflects blood glucose levels for the prior 3 months. Although a fasting blood glucose test and a 6-hour glucose tolerance test yield information about a client's use of glucose, the results are influenced by such factors as whether the client recently ate breakfast. Presence of ketones in the urine also provides information about glucose use but is limited in its diagnostic significance.

A client with a tentative diagnosis of hyperosmolar hyperglycemic nonketotic syndrome (HHNS) has a history of type 2 diabetes that is being controlled with an oral diabetic agent, tolazamide. Which laboratory test is the most important for confirming this disorder? -Serum potassium level -Serum sodium level -Arterial blood gas (ABG) values -Serum osmolarity

-Serum osmolarity Serum osmolarity is the most important test for confirming HHNS; it's also used to guide treatment strategies and determine evaluation criteria. A client with HHNS typically has a serum osmolarity of more than 350 mOsm/L. Serum potassium, serum sodium, and ABG values are also measured, but they aren't as important as serum osmolarity for confirming a diagnosis of HHNS. A client with HHNS typically has hypernatremia and osmotic diuresis. ABG values reveal acidosis, and the potassium level is variable.

Insulin is a hormone secreted by the Islets of Langerhans and is essential for the metabolism of carbohydrates, fats, and protein. The nurse understands the physiologic importance of gluconeogenesis, which refers to the: -Transport of potassium. -Release of glucose. -Synthesis of glucose from noncarbohydrate sources. -Storage of glucose as glycogen in the liver.

-Synthesis of glucose from noncarbohydrate sources. Gluconeogenesis refers to the making of glucose from noncarbohydrates. This occurs mainly in the liver. Its purpose is to maintain the glucose level in the blood to meet the body's demands.

The pancreas continues to release a small amount of basal insulin overnight, while a person is sleeping. The nurse knows that if the body needs more sugar: -The pancreatic hormone glucagon will stimulate the liver to release stored glucose. -Insulin will be released to facilitate the transport of sugar. -Glycogenesis will be decreased by the liver. -The process of gluconeogenesis will be inhibited.

-The pancreatic hormone glucagon will stimulate the liver to release stored glucose. When sugar levels are low, glucagon promotes hyperglycemia by stimulating the release of stored glucose. Glycogenolysis and gluconeogenesis will both be increased. Insulin secretion would promote hypoglycemia.

A client with a history of type 1 diabetes is demonstrating fast, deep, labored breathing and has fruity odored breath. What could be the cause of the client's current serious condition? -ketoacidosis -hyperosmolar hyperglycemic nonketotic syndrome -hepatic disorder -All options are correct.

-ketoacidosis Kussmaul respirations (fast, deep, labored breathing) are common in ketoacidosis. Acetone, which is volatile, can be detected on the breath by its characteristic fruity odor. If treatment is not initiated, the outcome of ketoacidosis is circulatory collapse, renal shutdown, and death. Ketoacidosis is more common in people with diabetes who no longer produce insulin, such as those with type 1 diabetes. People with type 2 diabetes are more likely to develop hyperosmolar hyperglycemic nonketotic syndrome because with limited insulin, they can use enough glucose to prevent ketosis but not enough to maintain a normal blood glucose level.

An agitated, confused client arrives in the emergency department. The client's history includes type 1 diabetes, hypertension, and angina pectoris. Assessment reveals pallor, diaphoresis, headache, and intense hunger. A stat blood glucose sample measures 42 mg/dl, and the client is treated for an acute hypoglycemic reaction. After recovery, the nurse teaches the client to treat hypoglycemia by ingesting: A. 10 to 15 g of a simple carbohydrate. B. 25 to 30 g of a simple carbohydrate. C. 2 to 5 g of a simple carbohydrate. D. 18 to 20 g of a simple carbohydrate.

10 to 15 g of a simple carbohydrate To reverse hypoglycemia, the American Diabetes Association recommends ingesting 10 to 15 g of a simple carbohydrate, such as three to five pieces of hard candy, two to three packets of sugar (4 to 6 tsp), or 4 oz of fruit juice. Then the client should check his blood glucose after 15 minutes. If necessary, this treatment may be repeated in 15 minutes. Ingesting only 2 to 5 g of a simple carbohydrate may not raise the blood glucose level sufficiently. Ingesting more than 15 g may raise it above normal, causing hyperglycemia.

The nurse is administering lispro insulin. Based on the onset of action, how long before breakfast should the nurse administer the injection? A. 10 to 15 minutes B. 30 to 40 minutes C. 1 to 2 hours D. 3 hours

10 to 15 minutes The onset of action of rapid-acting lispro insulin is within 10 to 15 minutes. It is used to rapidly reduce the glucose level.

A 6 months' pregnant patient was evaluated for gestational diabetes mellitus. The doctor considered prescribing insulin based on the serum glucose result of: A. 120 mg/dL, 1 hour postprandial. B. 138 mg/dL, 2 hours postprandial. C. 90 mg/dL before meals. D. 80 mg/dL, 1 hour postprandial.

138 mg/dL, 2 hours postprandial The goals for a 2-hour, postprandial blood glucose level are less than 120 mg/dL in a patient who might develop gestational diabetes.

A nurse is teaching a client with diabetes mellitus about self-management of his condition. The nurse should instruct the client to administer 1 unit of insulin for every: A. 10 g of carbohydrates. B. 15 g of carbohydrates. C. 20 g of carbohydrates. D. 25 g of carbohydrates.

15 g of carbohydrates The nurse should instruct the client to administer 1 unit of insulin for every 15 g of carbohydrates.

A client who is suspected of having diabetes is undergoing a postprandial glucose test. Which result would the nurse interpret as suggestive of diabetes? a) 70 mg /dL b) 110 mg/dL c) 220 mg/dL d) 160 mg/dL

160 mg/dL

What is the duration of regular insulin? 4 to 6 hours 3 to 5 hours 12 to 16 hours 24 hours

4 to 6 hours The duration of regular insulin is 4 to 6 hours; 3 to 5 hours is the duration for rapid-acting insulin such as Novolog. The duration of NPH insulin is 12 to 16 hours. The duration of Lantus insulin is 24 hours.

What is the duration of regular insulin? A. 12 to 16 hours B. 24 hours C. 4 to 6 hours D. 3 to 5 hours

4 to 6 hours The duration of regular insulin is 4 to 6 hours; 3 to 5 hours is the duration for rapid-acting insulin such as Novolog. The duration of NPH insulin is 12 to 16 hours. The duration of Lantus insulin is 24 hours.

A client with diabetes comes to the clinic for a follow-up visit. The nurse reviews the client's glycosylated hemoglobin test results. Which result would indicate to the nurse that the client's blood glucose level has been well-controlled? A. 7.5 % B. 8.0% C. 6.5% D. 8.5%

6.5% Normally the level of glycosylated hemoglobin is less than 7%. Thus a level of 6.5% would indicate that the client's blood glucose level is well-controlled. According to the American Diabetes Association, a glycosylated hemoglobin of 7% is equivalent to an average blood glucose level of 150 mg/dL. Thus, a level of 7.5% would indicate less control. Amount of 8% or greater indicate that control of the client's blood glucose level has been inadequate during the previous 2 to 3 months.

A client is receiving insulin lispro at 7:30 AM. The nurse ensures that the client has breakfast by which time? a. 7:45 AM b. 8:00 AM c. 8:15 AM d. 8:30 AM

7:45 AM Insulin lispro has an onset of 5 to 15 minutes. Therefore, the nurse would need to ensure that the client has his breakfast by 7:45 AM at the latest. Otherwise, the client may experience hypoglycemia.

Which type of insulin acts most quickly? 1. Glargine 2. Lispro 3. NPH 4. Regular

Correct response: Lispro Explanation: The onset of action of rapid-acting lispro is within 10 to 15 minutes. The onset of action of short-acting regular insulin is 30 minutes to 1 hour. The onset of action of intermediate-acting NPH insulin is 3 to 4 hours. The onset of action of very long-acting glargine is ~6 hours.

When administering insulin to a client with type 1 diabetes, which of the following would be most important for the nurse to keep in mind? A. Technique for injecting B. Accuracy of the dosage C. Duration of the insulin D. Area for insulin injection

Accuracy of the dosage The measurement of insulin is most important and must be accurate because clients may be sensitive to minute dose changes. The duration, area, and technique for injecting should also to be noted.

A hospitalized client is found to be comatose and hypoglycemic with a blood sugar of 50 mg/dL. Which of the following would the nurse do first? A. Check the client's urine for the presence of sugar and acetone. B. Administer 50% glucose intravenously. C. Encourage the client to drink orange juice with added sugar. D. Infuse 1000 mL D5W over a 12-hour period.

Administer 50% glucose intravenously. The unconscious, hypoglycemic client needs immediate treatment with IV glucose. If the client does not respond quickly and the blood glucose level continues to be low, glucagon, a hormone that stimulates the liver to release glycogen, or 20 to 50 mL of 50% glucose is prescribed for IV administration. A dose of 1,000 mL D5W over a 12-hour period indicates a lower strength of glucose and a slow administration rate. Checking the client's urine for the presence of sugar and acetone is incorrect because a blood sample is easier to collect and the blood test is more specific and reliable. An unconscious client cannot be given a drink. In such a case glucose gel may be applied in the buccal cavity of the mouth.

A client with type 1 diabetes presents with a decreased level of consciousness and a fingerstick glucose level of 39 mg/dl. His family reports that he has been skipping meals in an effort to lose weight. Which nursing intervention is most appropriate? A. Administering 1 ampule of 50% dextrose solution, per physician's order B. Administering a 500-ml bolus of normal saline solution C. Inserting a feeding tube and providing tube feedings D. Observing the client for 1 hour, then rechecking the fingerstick glucose level

Administering 1 ampule of 50% dextrose solution, per physician's order The nurse should administer 50% dextrose solution to restore the client's physiological integrity. Feeding through a feeding tube isn't appropriate for this client. A bolus of normal saline solution doesn't provide the client with the much-needed glucose. Observing the client for 1 hour delays treatment. The client's blood glucose level could drop further during this time, placing him at risk for irreversible brain damage.

A nurse is caring for a diabetic patient with a diagnosis of nephropathy. What would the nurse expect the urinalysis report to indicate? A. White blood cells B. Red blood cells C. Bacteria D. Albumin

Albumin Albumin is one of the most important blood proteins that leak into the urine. Although small amounts may leak undetected for years, its leakage into the urine is among the earliest signs that can be detected. Clinical nephropathy eventually develops in more than 85% of people with microalbuminuria but in fewer than 5% of people without microalbuminuria. The urine should be checked annually for the presence of microalbumin.

A client has been diagnosed with prediabetes and discusses treatment strategies with the nurse. What can be the consequences of untreated prediabetes? A. type 2 diabetes B. cardiac disease C. CVA D. All options are correct.

All options are correct The NIDDK has developed criteria that identify people with prediabetes, which can lead to type 2 diabetes, heart disease, and stroke.

A patient is admitted to the health care center with abdominal pain, nausea, and vomiting. The medical reports indicate a history of type 1 diabetes. The nurse suspects the patient's symptoms to be that of diabetic ketoacidosis (DKA). Which of the following actions will help the nurse confirm the diagnosis? a) Assessing for excessive sweating b) Assessing the patient's ability to move all extremities c) Assessing the patient's breath odor d) Assessing the patient's ability to take a deep breath

Assessing the patient's breath odor

Insulin is secreted by which of the following types of cells?

Beta cells

Which clinical characteristic is associated with type 2 diabetes (previously referred to as non-insulin-dependent diabetes mellitus)? A. Clients demonstrate islet cell antibodies B. Blood glucose can be controlled through diet and exercise C. Client is usually thin at diagnosis D. Client is prone to ketosis

Blood glucose can be controlled through diet and exercise Oral hypoglycemic agents may improve blood glucose concentrations if dietary modification and exercise are unsuccessful. Individuals with type 2 diabetes are usually obese at diagnosis. Individuals with type 2 diabetes rarely demonstrate ketosis, except with stress or infection. Individuals with type 2 diabetes do not demonstrate islet cell antibodies.

A client is admitted with hyperosmolar hyperglycemic nonketotic syndrome (HHNS). Which laboratory finding should the nurse expect in this client? A. Blood urea nitrogen (BUN) 15 mg/dl B. Arterial pH 7.25 C. Plasma bicarbonate 12 mEq/L D. Blood glucose level 1,100 mg/dl

Blood glucose level 1,100 mg/dl HHNS occurs most frequently in older clients. It can occur in clients with either type 1 or type 2 diabetes mellitus but occurs most commonly in those with type 2. The blood glucose level rises to above 600 mg/dl in response to illness or infection. As the blood glucose level rises, the body attempts to rid itself of the excess glucose by producing urine. Initially, the client produces large quantities of urine. If fluid intake isn't increased at this time, the client becomes dehydrated, causing BUN levels to rise. Arterial pH and plasma bicarbonate levels typically remain within normal limits.

Which clinical manifestation of type 2 diabetes occurs if glucose levels are very high? a. Hyperactivity b. Blurred vision c. Oliguria d. Increased energy

Blurred vision Blurred vision occurs when blood glucose levels are very high. The other clinical manifestations are not consistent with type 2 diabetes.

A client's blood glucose level is 45 mg/dl. The nurse should be alert for which signs and symptoms? a) Polyuria, polydipsia, hypotension, and hypernatremia b) Coma, anxiety, confusion, headache, and cool, moist skin c) Polyuria, polydipsia, polyphagia, and weight loss d) Kussmaul's respirations, dry skin, hypotension, and bradycardia

Coma, anxiety, confusion, headache, and cool, moist skin Signs and symptoms of hypoglycemia (indicated by a blood glucose level of 45 mf/dl) include anxiety, restlessness, headache, irritability, confusion, diaphoresis, cool skin, tremors, coma, and seizures. Kussmaul's respirations, dry skin, hypotension, and bradycardia are signs of diabetic ketoacidosis. Excessive thirst, hunger, hypotension, and hypernatremia are symptoms of diabetes insipidus. Polyuria, polydipsia, polyphagia, and weight loss are classic signs and symptoms of diabetes mellitus.

Which is not a cause of DKA? A. Illness or infection B. Competency in injecting insulin C. Undiagnosed and untreated diabetes D. Decreased or missed dose of insulin

Competency in injecting insulin Being able to competently inject insulin is not a cause of DKA. Undiagnosed and untreated diabetes decreased or missed dose of insulin, and illness or infection are potential causes of DKA.

A nurse educates a group of clients with diabetes mellitus on the prevention of diabetic nephropathy. Which of the following suggestions would be most important? A. Control blood glucose levels. B. Eat a high-fiber diet. C. Drink plenty of fluids. D. Take the antidiabetic drugs regularly.

Control blood glucose levels. Controlling blood glucose levels and any hypertension can prevent or delay the development of diabetic nephropathy. Drinking plenty of fluids does not prevent diabetic nephropathy. Taking antidiabetic drugs regularly may help to control blood glucose levels, but it is the control of these levels that is most important. A high-fiber diet is unrelated to the development of diabetic nephropathy.

Which of the following is an age-related change that may affect diabetes? Select all that apply. 1. Decreased renal function 2. Increased proprioception 3. Decreased vision 4. Increased bowel motility 5. Taste changes

Correct response: Decreased renal function Taste changes Decreased vision Explanation: Age-related changes include decreased renal function, taste changes, decreased vision, decreased bowel motility, and decreased proprioception.

When referred to a podiatrist, a client newly diagnosed with diabetes mellitus asks, "Why do you need to check my feet when I'm having a problem with my blood sugar?" The nurse's most helpful response to this statement is: 1. "Diabetes can affect sensation in your feet and you can hurt yourself without realizing it." 2. "It's easier to get foot infections if you have diabetes." 3. "The physician wants to be sure your shoes fit properly so you won't develop pressure sores." 4. "The circulation in your feet can help us determine how severe your diabetes is."

Correct response: "Diabetes can affect sensation in your feet and you can hurt yourself without realizing it." Explanation: The nurse should make the client aware that diabetes affects sensation in the feet and that he might hurt his foot but not feel the wound. Although it's important that the client's shoes fit properly, this isn't the only reason the client's feet need to be checked. Telling the client that diabetes mellitus increases the risk of infection or stating that the circulation in the client's feet indicates the severity of his diabetes doesn't provide the client with complete information.

A client with an amputation is learning how to apply a prosthetic limb. Which statement(s) made by the client indicates an increased risk for skin impairment? Select all that apply. 1. "I can wear a cotton garment with seams over the stump." 2. 'I will make sure the device is supportive but not too snug." 3. "I will make sure the padding is all placed in the front of the stump." 4. "I don't like wearing the prosthesis, but it helps me to walk." 5. "I can clean and inspect the skin of my amputated leg weekly."

Correct response: "I can clean and inspect the skin of my amputated leg weekly." "I will make sure the padding is all placed in the front of the stump." "I can wear a cotton garment with seams over the stump." Explanation: The client with a prosthetic limb would want to clean and inspect the skin of the amputated limb daily to ensure skin integrity is maintained. Having the padding of the device distributed evenly can help prevent pressure on the skin with the device. Wearing a cotton garment between the skin and prosthesis that does not have seams also helps decrease pressure and friction on the limb. The client would not want the device to fit too tightly as this can create pressure on the skin that could lead to skin breakdown. While the client needs emotional support with the use of prosthetic limbs, this is not part of skin protection and prevention of skin breakdown.

A client is taking glyburide (DiaBeta), 1.25 mg P.O. daily, to treat type 2 diabetes. Which statement indicates the need for further client teaching about managing this disease? 1. "I always carry hard candy to eat in case my blood sugar level drops." 2. "I avoid exposure to the sun as much as possible." 3. "I skip lunch when I don't feel hungry." 4. "I always wear my medical identification bracelet."

Correct response: "I skip lunch when I don't feel hungry." Explanation: The client requires further teaching if he states that he skips meals. A client who is receiving an oral antidiabetic agent should eat meals on a regular schedule because skipping a meal increases the risk of hypoglycemia. Carrying hard candy, avoiding exposure to the sun, and always wearing a medical identification bracelet indicate effective teaching.

Which statement best indicates that a client understands how to administer his own insulin injections? 1. "I need to wash my hands before I give myself my injection." 2. "I wrote down the steps in case I forget what to do." 3. "I need to be sure no air bubbles remain." 4. "If I'm not feeling well, I can get a friend or neighbor to help me."

Correct response: "I wrote down the steps in case I forget what to do." Explanation: The fact that the client has written down each step of insulin administration provides the best assurance that he'll follow through with all the proper steps. Awareness of air bubbles and hand washing indicate that the client understands certain aspects of giving an injection, but doesn't confirm he understands all of the steps. Saying that he can ask a friend or neighbor for help indicates a need for further instruction.

A nurse is preparing to administer insulin to a child who's just been diagnosed with type 1 diabetes. When the child's mother stops the nurse in the hall, she's crying and anxious to talk about her son's condition. The nurse's best response is: 1. "Everything will be just fine. I'll be back in a minute and then we can talk." 2. "I can't talk now. I have to give your son his insulin as soon as possible." 3. "If you'll wait in your son's room, the physician will talk with you as soon as he's free." 4. "I'm going to give your son some insulin. Then I'll be happy to talk with you."

Correct response: "I'm going to give your son some insulin. Then I'll be happy to talk with you." Explanation: Attending to the mother's needs is a critical part of caring for a sick child. In this case however, administering insulin in a prompt manner supersedes the mother's needs. By informing the mother that she's going to administer the insulin and will then make time to talk with her, the nurse recognizes the mother's needs as legitimate. She provides a reasonable response while attending to the priority of administering insulin as soon as possible. Telling the mother that she can't talk with her or telling her to wait for the physician could increase the mother's fear and anxiety. The nurse shouldn't tell the mother that everything will be fine; the nurse doesn't know that everything will be fine.

The nurse is teaching an adolescent with type 1 diabetes about signs and symptoms of hypoglycemia. Which of the following statements by the client help the nurse determine that the teaching has been effective? Select all that apply. 1. "If my blood sugar is low, my blood pressure will increase." 2. "If my blood sugar is low, I may feel sweaty and anxious." 3. "If my blood sugar is low, my heart rate will speed up." 4. "If my blood sugar is low, my breath will smell fruity." 5. "If my blood sugar is low, I will be very thirsty."

Correct response: "If my blood sugar is low, I may feel sweaty and anxious." "If my blood sugar is low, my heart rate will speed up." "If my blood sugar is low, my blood pressure will increase." Explanation: Sweating, anxiety, tachycardia, and increased blood pressure are signs of hypoglycemia. Thirst and a fruity breath odor are signs of hyperglycemia.

A client with type 1 diabetes has a highly elevated glycosylated hemoglobin (Hb) test result. In discussing the result with the client, the nurse is most accurate in stating: 1. "It looks like you aren't following the ordered diabetic diet." 2. "It tells us about your sugar control for the last 3 months." 3. "The test must be repeated following a 12-hour fast." 4. "Your insulin regimen must be altered significantly."

Correct response: "It tells us about your sugar control for the last 3 months." Explanation: The nurse is providing accurate information to the client when she states that the glycosylated Hb test provides an objective measure of glycemic control over a 3-month period. The test helps identify trends or practices that impair glycemic control, and it doesn't require a fasting period before blood is drawn. The nurse can't conclude that the result occurs from poor dietary management or inadequate insulin coverage.

A nurse prepares teaching for a client with newly-diagnosed diabetes. Which statements about the role of insulin will the nurse include in the teaching? Select all that apply. 1. "Insulin interferes with the release of growth hormone from the pituitary." 2. "Insulin permits entry of glucose into the cells of the body." 3. "Insulin promotes the storage of fat in adipose tissue." 4. "Insulin interferes with glucagon from the pancreas." 5. "Insulin promotes synthesis of proteins in various body tissues."

Correct response: - "Insulin permits entry of glucose into the cells of the body." - "Insulin promotes synthesis of proteins in various body tissues." - "Insulin promotes the storage of fat in adipose tissue." Explanation: Insulin is a hormone secreted by the endocrine part of the pancreas. In addition to lowering blood glucose by permitting entry of glucose into the cells, insulin also promotes protein synthesis and the storage of fat in adipose tissue. Somatostatin exerts a hypoglycemic effect by interfering with glucagon from the pancreas and the release of growth hormone from the pituitary.

A nurse is preparing a continuous insulin infusion for a child with diabetic ketoacidosis and a blood glucose level of 800 mg/dl. Which solution is the most appropriate at the beginning of therapy? 1. 100 units of neutral protamine Hagedorn (NPH) insulin in normal saline solution 2. 100 units of regular insulin in normal saline solution 3. 100 units of NPH insulin in dextrose 5% in water 4. 100 units of regular insulin in dextrose 5% in water

Correct response: 100 units of regular insulin in normal saline solution Explanation: Continuous insulin infusions use only short-acting regular insulin. Insulin is added to normal saline solution and administered until the client's blood glucose level falls. Further along in the therapy, a dextrose solution is administered to prevent hypoglycemia.

A nurse knows to assess a patient with type 1 diabetes for postprandial hyperglycemia. The nurse knows that glycosuria is present when the serum glucose level exceeds: 1. 180 mg/dL 2. 120 mg/dL 3. 140 mg/dL 4. 160 mg/dL

Correct response: 180 mg/dL Explanation: Glycosuria occurs when the renal threshold for sugar exceeds 180 mg/dL. Glycosuria leads to an excessive loss of water and electrolytes (osmotic diuresis)

When the dawn phenomenon occurs, the patient has relatively normal blood glucose until approximate what time of day? 1. 9 AM 2. 5 AM 3. 7 AM 4. 3 AM

Correct response: 3 AM Explanation: During the dawn phenomenon, the patient has a relatively normal blood glucose level until about 3 AM, when the level begins to rise.

Glycosylated hemoglobin reflects blood glucose concentrations over which period of time? 1. 1 month 2. 3 months 3. 6 months 4. 9 months

Correct response: 3 months Explanation: Glycosylated hemoglobin is a blood test that reflects average blood glucose concentrations over a period of 3 months.

A nurse administered neutral protamine Hagedorn (NPH) insulin to a client with diabetes mellitus at 7 a.m. (0700). At what time should the nurse expect the client to be most at risk for hypoglycemia? 1. noon (1200) 2. 10 a.m. (1000) 3. 4 p.m. (1600) 4. 10 p.m. (2200)

Correct response: 4 p.m. (1600) Explanation: NPH is an intermediate-acting insulin that peaks 8 to 12 hours after administration. Because the nurse administered NPH insulin at 7 a.m. (0700), the client is at greatest risk for hypoglycemia from 3 p.m. (1500) to 7 p.m. (1900).

A health care provider prescribes short-acting insulin for a patient, instructing the patient to take the insulin 20 to 30 minutes before a meal. The nurse explains to the patient that Humulin-R taken at 6:30 AM will reach peak effectiveness by: 1. 12:30 PM. 2. 2:30 PM. 3. 10:30 AM. 4. 8:30 AM.

Correct response: 8:30 AM. Explanation: Short-acting insulin reaches its peak effectiveness 2 to 3 hours after administration. See Table 30-3 in the text.

A client with type 1 diabetes is scheduled to receive 30 units of 70/30 insulin. There is no 70/30 insulin available. As a substitution, the nurse may give the client 1. 9 units regular insulin and 21 units neutral protamine Hagedorn (NPH). 2. 21 units regular insulin and 9 units NPH. 3. 20 units regular insulin and 10 units NPH. 4. 10 units regular insulin and 20 units NPH.

Correct response: 9 units regular insulin and 21 units neutral protamine Hagedorn (NPH). Explanation: A 70/30 insulin preparation is 70% NPH and 30% regular insulin. Therefore, a correct substitution requires mixing 21 units of NPH and 9 units of regular insulin. The other choices are incorrect dosages for the ordered insulin.

A client with diabetic ketoacidosis has been brought into the ED. Which intervention is not a goal in the initial medical treatment of diabetic ketoacidosis? 1. Monitor serum electrolytes and blood glucose levels. 2. Administer glucose. 3. Administer potassium replacements. 4. Administer isotonic fluid at a high volume.

Correct response: Administer glucose. Explanation: Insulin is given intravenously. Insulin reduces the production of ketones by making glucose available for oxidation by the tissues and by restoring the liver's supply of glycogen. As insulin begins to lower the blood glucose level, the IV solution is changed to include one with glucose. Periodic monitoring of serum electrolytes and blood glucose levels is necessary. Isotonic fluid is instilled at a high volume, for example, 250 to 500 mL/hour for several hours. The rate is adjusted once the client becomes rehydrated and diuresis is less acute. Potassium replacements are given despite elevated serum levels to raise intracellular stores.

A client with systemic lupus erythematosus (SLE) reports having hands that become pale, blue, and painful when exposed to the cold. Raynaud's disease is suspected. What nursing interventions should the nurse suggest to improve the signs and symptoms associated with this disease? Select all that apply. 1. Take the prescribed calcium channel blockers. 2. Avoid caffeine. 3. Limit exercise to affected extremities. 4. Stop smoking. 5. Learn biofeedback techniques.

Correct response: Avoid caffeine. Stop smoking. Take the prescribed calcium channel blockers. Learn biofeedback techniques. Explanation: Raynaud's disease results from reduced blood flow to the extremities, especially the hands and feet where blood vessels are smaller, when exposed to cold or stress. It is commonly associated with connective tissue disorders, such as SLE. Signs and symptoms include pallor, coldness, numbness, throbbing pain, and cyanosis. Interventions are tailored to control the signs and symptoms from the disease, such as avoiding triggers (like cold temperatures), reducing stress, and taking medications (such as calcium channel blockers, which relax and open up small blood vessels, alpha blockers, and vasodilators). Exercise is permitted to improve circulation. It is also important to avoid handling items that vibrate, such as tools. Hands and feet must be cared for and prevented from injury and compressive items, such as tight wristbands, rings, footwear. Caffeine should be avoided, as it causes blood vessels to narrow. Stopping smoking and avoiding secondhand smoke is also important, as it lowers skin temperature and causes blood vessels to constrict. Biofeedback techniques, such as guided imagery, can be learned to help with mind control over body temperature.

The nurse is reviewing the initial laboratory test results of a client diagnosed with DKA. Which of the following would the nurse expect to find? 1. Blood pH of 6.9 2. Blood glucose level of 250 mg/dL 3. Serum bicarbonate of 19 mEq/L 4. PaCO2 of 40 mm Hg

Correct response: Blood pH of 6.9 Explanation: With DKA, blood glucose levels are elevated to 300 to 1000 mg/dL or more. Urine contains glucose and ketones. The blood pH ranges from 6.8 to 7.3. The serum bicarbonate level is decreased to levels from 0 to 15 mEq/L. The compensatory breathing pattern can lower the partial pressure of carbon dioxide in arterial blood (PaCO2) to levels of 10 to 30 mm Hg.

A nurse is teaching a client about insulin infusion pump use. What interventions should the nurse include to prevent infection at the injection site? Select all that apply. 1. Wear sterile gloves when inserting the needle. 2. Use sterile techniques when changing the needle. 3. Change the needle every 3 days. 4. Cleanse the skin at the insertion site for 15 seconds using alcohol. 5. Take the ordered antibiotics before initiating treatment.

Correct response: Change the needle every 3 days. Use sterile techniques when changing the needle. Cleanse the skin at the insertion site for 15 seconds using alcohol. Explanation: The nurse should teach the client to change the needle every 3 days to prevent infection, use sterile techniques, and properly cleanse the skin. The client does not need to wear gloves when inserting the needle. Antibiotic therapy is not necessary before initiating treatment.

A client with a serum glucose level of 618 mg/dl (34.33 mmol/L) is admitted to the facility. The client is awake and oriented, has hot dry skin, and has the following vital signs: temperature of 100.6° F (38.1° C), heart rate of 116 beats/minute, and blood pressure of 108/70 mm Hg. Based on these assessment findings, which nursing diagnosis takes highest priority? 1. Deficient fluid volume related to osmotic diuresis 2. Ineffective thermoregulation related to dehydration 3. Imbalanced nutrition: Less than body requirements related to insulin deficiency 4. Decreased cardiac output related to elevated heart rate

Correct response: Deficient fluid volume related to osmotic diuresis Explanation: A serum glucose level of 618 mg/dl (34.33 mmol/L) indicates hyperglycemia, which causes polyuria and fluid volume deficit, making Deficient fluid volume related to osmotic diuresis the highest priority. In this client, tachycardia is more likely to result from fluid volume deficit than from decreased cardiac output because the client's blood pressure is normal. Although the client's serum glucose is elevated, food isn't a priority because fluids and insulin should be administered to lower the serum glucose level. Therefore, a diagnosis of Imbalanced nutrition: Less than body requirements isn't appropriate. A temperature of 100.6° F (38.1° C) isn't life-threatening, eliminating Ineffective thermoregulation as the top priority.

A client with long-standing type 1 diabetes is admitted to the hospital with unstable angina pectoris. After the client's condition stabilizes, the nurse evaluates the diabetes management regimen. The nurse learns that the client sees the physician every 4 weeks, injects insulin after breakfast and dinner, and measures blood glucose before breakfast and at bedtime. Consequently, the nurse should formulate a nursing diagnosis of: 1. Health-seeking behaviors (diabetes control). 2. Deficient knowledge (treatment regimen). 3. Defensive coping. 4. Impaired adjustment.

Correct response: Deficient knowledge (treatment regimen). Explanation: The client should inject insulin before, not after, breakfast and dinner — 30 minutes before breakfast for the a.m. dose and 30 minutes before dinner for the p.m. dose. Therefore, the client has a knowledge deficit regarding when to administer insulin. By taking insulin, measuring blood glucose levels, and seeing the physician regularly, the client has demonstrated the ability and willingness to modify his lifestyle as needed to manage the disease. This behavior eliminates the nursing diagnoses of Impaired adjustment and Defensive coping. Because the nurse, not the client, questioned the client's health practices related to diabetes management, the nursing diagnosis of Health-seeking behaviors isn't warranted.

A nurse is caring for a client with an abnormally low blood glucose concentration. What glucose level should the nurse observe when assessing laboratory results? Between 70 and 75 mg/dL (3.9 to 4.16 mmol/L) Less than 70 mg/dL (3.7 mmol/L) Between 75 and 90 mg/dL (4.16 to 5.00 mmol/L) 95 mg/dL (5.27 mmol/L)

Correct response: Less than 70 mg/dL (3.7 mmol/L) Explanation: Hypoglycemia means low (hypo) sugar in the blood (glycemia), and occurs when the blood glucose level falls to less than 70 mg/dL (3.7 mmol/L). Severe hypoglycemia is when glucose levels are less the 40 mg/dL (2.5 mmol/L).

Which of the following outcomes is desired when a client with arterial insufficiency has poor tissue perfusion in the extremities? Select all that apply. 1. Decreased muscle pain with activity. 2. Lungs clear to auscultation. 3. Improved respiratory status. 4. Participation in self-care measures. 5. Extremities warm to touch.

Correct response: Extremities warm to touch. Decreased muscle pain with activity. Explanation: The desired outcome for the client with poor circulation to the extremities is evidence of adequate blood flow to the area. The temperature of the involved extremity is an important indicator for a client with peripheral vascular disease. The temperature will indicate the degree to which the blood supply is getting to the extremity. Warmth indicates adequate blood flow. Pain is also an indicator of blood flow. Pain, such as muscle pain, suggests ischemia and lack of oxygen that results when the oxygen demand becomes greater than the supply. Thus, a decrease in muscle pain with activity would suggest improvement in blood flow to the area. Improved respiratory status and clear lungs are unrelated to the poor tissue perfusion. Although participation in self-care measures is always helpful, this outcome is not a result of establishing circulation to the extremities.

A client with type 1 diabetes mellitus is receiving short-acting insulin to maintain control of blood glucose levels. In providing glucometer instructions, the nurse would instruct the client to use which site for most accurate findings? 1. Upper arm 2. Forearm 3. Thigh 4. Finger

Correct response: Finger Explanation: Even though the fingertips have a higher number of nerve endings, this site provides the most accurate blood sugar reading. Alternate sites, such as upper arm, forearm, and thighs are regarded as lagging test sites and are not an option for people who require tight glucose control.

Which is the best nursing explanation for the symptom of polyuria in a client with diabetes mellitus? 1. High sugar pulls fluid into the bloodstream, which results in more urine production. 2. With diabetes, drinking more results in more urine production. 3. The body's requirement for fuel drives the production of urine. 4. Increased ketones in the urine promote the manufacturing of more urine.

Correct response: High sugar pulls fluid into the bloodstream, which results in more urine production. Explanation: The hypertonicity from concentrated amounts of glucose in the blood pulls fluid into the vascular system, resulting in polyuria. The urinary frequency triggers the thirst response, which then results in polydipsia. Ketones in the urine and body requirements do not affect the production of urine.

An older adult patient is in the hospital being treated for sepsis related to a urinary tract infection. The patient has started to have an altered sense of awareness, profound dehydration, and hypotension. What does the nurse suspect the patient is experiencing? 1. Diabetic ketoacidosis 2. Multiple-organ dysfunction syndrome 3. Systemic inflammatory response syndrome 4. Hyperglycemic hyperosmolar syndrome

Correct response: Hyperglycemic hyperosmolar syndrome Explanation: Hyperglycemic hyperosmolar syndrome (HHS) occurs most often in older people (50 to 70 years of age) who have no known history of diabetes or who have type 2 diabetes (Reynolds, 2012). The clinical picture of HHS is one of hypotension, profound dehydration (dry mucous membranes, poor skin turgor), tachycardia, and variable neurologic signs (e.g., alteration of consciousness, seizures, hemiparesis).

For a client with hyperglycemia, which assessment finding best supports a nursing diagnosis of Deficient fluid volume? 1. Increased urine osmolarity 2. Cool, clammy skin 3. Jugular vein distention 4. Decreased serum sodium level

Correct response: Increased urine osmolarity Explanation: In hyperglycemia, urine osmolarity (the measurement of dissolved particles in the urine) increases as glucose particles move into the urine. The client experiences glucosuria and polyuria, losing body fluids and experiencing deficient fluid volume. Cool, clammy skin; jugular vein distention; and a decreased serum sodium level are signs of fluid volume excess, the opposite imbalance.

The nurse is caring for a client with peripheral vascular disease (PVD). Which action would the nurse do to ensure an accurate assessment? 1. Keep the client uncovered. 2. Keep the client warm. 3. Maintain room temperature at 78°F (25.6°C). 4. Match the room temperature to the client's body temperature.

Correct response: Keep the client warm. Explanation: Vasodilation or vasoconstriction will affect the assessment findings in a client with PVD, so the nurse would keep the client warm. The nurse would keep the client covered and expose only the portion of the client's body that the nurse is assessing. The nurse would also keep the client warm by maintaining the room temperature between 68°F and 74°F (20° and 23.3°C). Extreme temperatures have a negative effect on clients with PVD. Keeping the client uncovered would cause the client to become chilled. Matching the room temperature to the client's body temperature is inappropriate.

The nurse suspects that a patient with diabetes has developed proliferative retinopathy. The nurse confirms this by the presence of which of the following diagnostic signs? 1. Neovascularization into the vitreous humor 2. Decreased capillary permeability 3. Microaneurysm formation 4. The leakage of capillary wall fragments into surrounding areas

Correct response: Neovascularization into the vitreous humor Explanation: Proliferative retinopathy, an ocular complication of diabetes, occurs because of the abnormal growth of new blood vessels on the retina that bleed into the vitreous and block light. Blood vessels in the vitreous form scar tissue that can pull and detach the retina. Neovascularization into the vitreous humor is considered a diagnostic sign.

A client with long-standing type 1 diabetes is admitted to the hospital with unstable angina pectoris and a hemoglobin A1C of 10%. After evaluating the client's diabetic management regimen the nurse should do what? Select all that apply. 1. Consult the diabetic educator. 2. Expect to give large doses of oral sulfonylureas. 3. Obtain a blood sample for a troponin level. 4. Connect to continuous cardiac monitoring and prepare to cardiovert. 5. Prepare to schedule a trans-esophageal echo (TEE).

Correct response: Prepare to schedule a trans-esophageal echo (TEE). Consult the diabetic educator. Obtain a blood sample for a troponin level. Explanation: Consulting the diabetic educator will insure the client receives a comprehensive education about diabetes. By checking the troponin, this identifies possible MI and is needed since the client indicates chest pain. Oral hypergylcemics are not effective with type 1 diabetes. The TEE may be indicated since the angina is unstable.

Which is the primary reason for encouraging injection site rotation in an insulin dependent diabetic? 1. Prevent muscle destruction. 2. Promote absorption. 3. Minimize discomfort. 4. Avoid infection.

Correct response: Promote absorption. Explanation: Subcutaneous injection sites require rotation to avoid breakdown and/or buildup of subcutaneous fat, either of which can interfere with insulin absorption in the tissue. Infection and discomfort are risks involved with injection site but not the primary reason for rotation of sites. Insulin is not injected into the muscle.

The nurse is explaining glycosylated hemoglobin testing to a diabetic client. Which of the following provides the best reason for this order? 1. Best indicator for the nutritional state of the client 2. Reflects the amount of glucose stored in hemoglobin over past several months. 3. Provides best information on the body's ability to maintain normal blood functioning 4. Is less costly than performing daily blood sugar test

Correct response: Reflects the amount of glucose stored in hemoglobin over past several months. Explanation: Hemoglobin A1c tests reflect the amount of glucose that is stored in the hemoglobin molecule during its life span of 120 days. This test provides a more accurate picture of overall glucose control in a client. Glycosylated hemoglobin test does not indicate normal blood functioning or nutritional state of the client. Self-monitoring with a glucometer is still encouraged in clients who are taking insulin or have unstable blood glucose levels.

A client has been diagnosed with Raynaud's phenomenon on the tip of the nose and fingertips. The health care provider has prescribed reserpine to determine if the client will obtain relief. The client often works outside in cold weather and also smokes two packs of cigarettes per day. The nurse should include which instructions in the discharge plan for this client? Select all that apply. 1. Stop smoking. 2. Wear a face covering and gloves in the winter. 3. Report signs of orthostatic hypotension. 4. Place fingertips in cool water to rewarm them. 5. Find employment that can be done in a warm environment.

Correct response: Stop smoking. Wear a face covering and gloves in the winter. Report signs of orthostatic hypotension. Explanation: Vasospastic disorder (Raynaud's disease) is a form of intermittent arteriolar vasoconstriction that results in coldness, pain, and pallor of the fingertips, toes, or tip of the nose, and a rebound circulation with redness and pain. The nurse should instruct the client to stop smoking because nicotine is a vasoconstrictor. An adverse effect of reserpine is orthostatic hypotension. The client should report dizziness and low blood pressure as it may be necessary to consider stopping the drug. The client should prevent vasoconstriction by covering affected parts when in cold environments. The nurse can teach the client to rewarm exposed extremities by using warm water or placing them next to the body, such as under the axilla. It is not realistic to ask this client to change jobs at this time.

A nurse educator been invited to local seniors center to discuss health-maintaining strategies for older adults. The nurse addresses the subject of diabetes mellitus, its symptoms, and consequences. What should the educator teach the participants about type 1 diabetes? 1. Type 1 diabetes always develops before the age of 20. 2. New cases of diabetes are highly uncommon in older adults. 3. The participants are unlikely to develop a new onset of type 1 diabetes. 4. New cases of diabetes will be split roughly evenly between type 1 and type 2.

Correct response: The participants are unlikely to develop a new onset of type 1 diabetes. Explanation: Type 1 diabetes usually (but not always) develops in people younger than 20. In older adults, an onset of type 2 is far more common. A significant number of older adults develops type 2 diabetes.

A client receives a daily injection of glargine insulin at 7:00 a.m. When should the nurse monitor this client for a hypoglycemic reaction? 1. Between 8:00 and 10:00 a.m. 2. Between 4:00 and 6:00 p.m. 3. Between 7:00 and 9:00 p.m. 4. This insulin has no peak action and does not cause a hypoglycemic reaction.

Correct response: This insulin has no peak action and does not cause a hypoglycemic reaction. Explanation: "Peakless" basal or very long-acting insulins are approved by the U.S. Food and Drug Administration for use as a basal insulin; that is, the insulin is absorbed very slowly over 24 hours and can be given once a day. It has is no peak action.

The nurse understands that a client with diabetes mellitus is at greater risk for developing which of the following complications? 1. Lifelong obesity 2. Elevated triglycerides 3. High blood pressure 4. Urinary tract infections

Correct response: Urinary tract infections Explanation: Elevated levels of blood glucose and glycosuria supports bacterial growth and places the diabetic at greater risk for urinary tract, skin, and vaginal infections. Obesity, elevated triglycerides, and high blood pressure are considered symptoms of metabolic syndrome, which can result in type 2 diabetes mellitus.

A client with type 2 diabetes mellitus needs instruction on proper foot care. Which instructions should the nurse include in client teaching? Select all that apply. 1. Go barefoot only when you know your home environment. 2. See a podiatrist regularly to have your feet checked. 3. Apply foot powder after bathing. 4. Use scissors to trim toenails. 5. Wear loose-fitting shoes. 6. Wear cotton socks.

Correct response: Wear cotton socks. Apply foot powder after bathing. See a podiatrist regularly to have your feet checked. Explanation: Foot care for a client with diabetes mellitus includes keeping the feet dry with the application of foot powder and wearing cotton socks to absorb moisture. The client should have a podiatrist check the feet regularly to detect problems early. To prevent injury to the feet, the client should be instructed not to cut the toenails with scissors, walk barefoot, or wear loose-fitting shoes.

A nurse is participating in a diabetes screening program. Which clients are at risk for developing type 2 diabetes? Select all that apply. 1. an 18-year-old immigrant from Mexico who jogs four times a week 2. a 44-year-old Native American (First Nations) person who has a body mass index (BMI) of 32 3. a 12-year-old who is overweight 4. a 55-year-old Asian who has hypertension and two siblings with type 2 diabetes 5. a 32-year-old female who delivered a 9.5-lb (4,309-g) infant

Correct response: a 32-year-old female who delivered a 9.5-lb (4,309-g) infant a 44-year-old Native American (First Nations) person who has a body mass index (BMI) of 32 a 55-year-old Asian who has hypertension and two siblings with type 2 diabetes a 12-year-old who is overweight Explanation: The risk factors for developing type 2 diabetes include giving birth to an infant weighing >9 lb (4,100 g); obesity (BMI over 30); ethnicity of Asian, African, Native American, or First Nations; age greater than 45 years; hypertension; and family history in parents or siblings. Childhood obesity is also a risk factor for type 2 diabetes. Maintaining an ideal weight, eating a low-fat diet, and exercising regularly decrease the risk of type 2 diabetes.

The nurse should review the glucose level of which clients who are going to surgery today? Select all that apply. 1. a client with a high stress response to surgery 2. a client receiving corticosteroids for the past 3 months 3. a client with a family history of diabetes receiving dextrose 5% in lactated Ringer's solution (DLR) IV fluids 4. a client with diabetes mellitus controlled by diet 5. a client who consumes a high carbohydrate diet

Correct response: a client with diabetes mellitus controlled by diet a client with a high stress response to surgery a client receiving corticosteroids for the past 3 months Explanation: Clients who have diabetes mellitus controlled by diet, those with a high stress response to surgery, or those who have been on steroid treatment for the last 3 months should have their serum glucose level assessed. A client with a family history of diabetes receiving D5LR IV fluids does not need to have the serum glucose level checked unless other clinical manifestations are present. The client who has a high carbohydrate diet should be able to metabolize the glucose unless there are other health problems.

When reviewing the urinalysis report of a client with newly diagnosed diabetes mellitus, the nurse would expect which urine characteristics to be abnormal? Select all that apply. 1. ketone bodies 2. pH 3. specific gravity 4. glucose level 5. amount 6. odor

Correct response: amount odor glucose level ketone bodies Explanation: Diabetes mellitus is associated with increased amounts of urine, a sweet or fruity odor, and glucose and ketone bodies in the urine. It does not affect the urine's pH or specific gravity.

What should the nurse recognize as the first sign of peripheral arterial disease (PAD) in inactive older adults? 1. dry, shiny skin of the extremities 2. gangrene 3. pain in the extremities 4. diminished pulses

Correct response: gangrene Explanation: In older adults who are inactive, limb ischemia or gangrene may be the first sign of PAD because these clients manage their lifestyle by adjusting for limitations imposed by comorbidity; therefore, not walking far enough to develop the pain of claudication. Even though diminished pulses and dry, shiny skin may be present, this is not apparent until trauma occurs and gangrene develops.

A client with type 1 diabetes asks the nurse about taking an oral antidiabetic agent. The nurse explains that these medications are effective only if the client: 1. has type 2 diabetes. 2. has type 1 diabetes. 3. prefers to take insulin orally. 4. is pregnant and has type 2 diabetes.

Correct response: has type 2 diabetes. Explanation: Oral antidiabetic agents are effective only in adult clients with type 2 diabetes. Oral antidiabetic agents aren't effective in type 1 diabetes. Pregnant and lactating women aren't ordered oral antidiabetic agents because the effect on the fetus or breast-fed infant is uncertain.

A client with type 1 diabetes has been on a regimen of multiple daily injection therapy. The client is being converted to continuous subcutaneous insulin therapy. While teaching continuous subcutaneous insulin therapy, the nurse should tell the client that the regimen includes the use of: 1. short- and long-acting insulins. 2. intermediate- and long-acting insulins. 3. rapid-acting insulin only. 4. short- and intermediate-acting insulins.

Correct response: rapid-acting insulin only. Explanation: A continuous subcutaneous insulin regimen uses a basal rate and boluses of rapid-acting insulin. Multiple daily injection therapy uses a combination of rapid-acting and intermediate- or long-acting insulins.

The nurse is obtaining a health history from a client with diabetes mellitus who has been taking insulin for 20 years. Currently, the client reports having periods of hypoglycemia followed by periods of hyperglycemia. What should the nurse ask about the client's current management plan? Is the client: 1. injecting insulin at a site of lipodystrophy? 2. eating snacks between meals? 3. using an insulin pump? 4. adjusting insulin according to blood glucose levels?

Correct response: injecting insulin at a site of lipodystrophy? Explanation: Lipodystrophy, specifically lipohypertrophy, involves swelling of the fat at the site of repeated injections, which can interfere with the absorption of insulin, resulting in erratic blood glucose levels. Because the client has been receiving insulin for many years, this is the most likely cause of poor control. Eating snacks between meals causes hyperglycemia. Adjusting insulin according to blood glucose levels would not cause hypoglycemia but normal levels. Initiating an insulin pump would not, of itself, cause the periods of hyperglycemia.

A nurse is teaching a client about insulin therapy. The nurse knows the client needs additional teaching when they state that insulin may interact with 1. aspirin. 2. hormonal contraceptives. 3. metoprolol. 4. hydrochlorothiazide (Hydro DIURIL).

Correct response: metoprolol. Explanation: Although metoprolol may mask the signs of hypoglycemia, it doesn't interact with insulin. Therefore, the client requires additional teaching. Thiazide diuretics such as hydrochlorothiazide, aspirin, and hormonal contraceptives all interact with insulin.

A nurse assesses a 40-year-old female client with Raynaud's phenomenon involving her right hand. The nurse records the information in the progress notes, as shown. From these findings, what should the nurse first instruct the client how to manage? 1. numbness. 2. acute pain. 3. potential for skin breakdown. 4. lack of circulation.

Correct response: numbness. Explanation: The client has numbness in the fingertips, and the nurse should first help the client regain sensory perception and discuss strategies for prevention of injury. The client does not have acute pain. The client does have adequate circulation and is not at risk for skin breakdown at this time.

A nurse administers glucagon to a client with diabetes mellitus, then monitors the client for adverse drug reactions and interactions. Which drug type interacts adversely with glucagon? 1. oral anticoagulants 2. anabolic steroids 3. beta-adrenergic blockers 4. thiazide diuretics

Correct response: oral anticoagulants Explanation: As a normal body protein, glucagon interacts adversely only with oral anticoagulants, increasing their anticoagulant effects. It doesn't interact adversely with anabolic steroids, beta-adrenergic blockers, or thiazide diuretics.

A client with insulin-dependent diabetes develops a seizure disorder and has been prescribed phenytoin. Which information should the nurse include in the teaching plan? The client should: (Select all that apply.) 1. substitute various brands of phenytoin as long as the dosage is the same. 2. use a soft toothbrush and floss the teeth daily. 3. take potassium supplements to prevent hypokalemia. 4. increase the daily calorie allotment by 200 calories. 5. report changes in blood glucose levels to the health care provider.

Correct response: report changes in blood glucose levels to the health care provider. use a soft toothbrush and floss the teeth daily. Explanation: With long-term use, phenytoin can cause gingival hyperplasia, so it is essential that the client understand how to provide proper oral hygiene. Phenytoin does not lead to the development of diabetes, but it can affect diabetics' blood glucose levels and require adjustment of their hypoglycemic agents.The client cannot substitute various brands of phenytoin because they are not bioequivalent.It is not necessary for the client to take potassium supplements.It is not necessary to change the diet plan when taking phenytoin.

The nurse is administering an insulin drip to a patient in ketoacidosis. What insulin does the nurse know can be used intravenously? 1. short-acting 2. long-acting 3. rapid acting 4. intermediate-acting

Correct response: short-acting Explanation: Insulins may be grouped into several categories based on the onset, peak, and duration of action. Short-acting, also known as regular insulin, is the only insulin administered by IV.

Which factors contribute to a risk for amputation in a client with peripheral vascular disease? Select all that apply. 1. work that requires prolonged standing 2. a serum cholesterol concentration of 275 mg/dL (15.3 mmol/L) 3. uncontrolled diabetes mellitus for 15 year 4. a hemoglobin level of 14.2 g/dL 5. a 20-pack-year history of cigarette smoking

Correct response: uncontrolled diabetes mellitus for 15 year a 20-pack-year history of cigarette smoking a serum cholesterol concentration of 275 mg/dL (15.3 mmol/L) Explanation: Uncontrolled diabetes mellitus is considered a risk factor for peripheral vascular disease (PVD) because of the macroangiopathic and microangiopathic changes that result from poor blood glucose control. Cigarette smoking is a known risk factor for peripheral vascular disease; nicotine is a potent vasoconstrictor. Serum cholesterol levels >200 mg/dL (11.1 mmol/L) are considered a risk factor for peripheral vascular disease. The hemoglobin level is within normal range and is not a risk for PVD. Prolonged standing is a risk factor for venous stasis and varicose veins.

A client is diagnosed with diabetes mellitus. Which assessment finding best supports a nursing diagnosis of Ineffective coping related to diabetes mellitus? A. Recent weight gain of 20 lb (9.1 kg) B. Failure to monitor blood glucose levels C. Crying whenever diabetes is mentioned D. Skipping insulin doses during illness

Crying whenever diabetes is mentioned A client who cries whenever diabetes is mentioned is demonstrating ineffective coping. A recent weight gain and failure to monitor blood glucose levels would support a nursing diagnosis of Noncompliance: Failure to adhere to therapeutic regimen. Skipping insulin doses during illness would support a nursing diagnosis of Deficient knowledge related to treatment of diabetes mellitus.

An obese Hispanic client, age 65, is diagnosed with type 2 diabetes. Which statement about diabetes mellitus is true? A. Nearly two-thirds of clients with diabetes mellitus are older than age 60. B. Diabetes mellitus is more common in Hispanics and Blacks than in Whites. C. Approximately one-half of the clients diagnosed with type 2 diabetes are obese. D. Type 2 diabetes mellitus is less common than type 1 diabetes mellitus.

Diabetes mellitus is more common in Hispanics and Blacks than in Whites Diabetes mellitus is more common in Hispanics and Blacks than in Whites. Only about one-third of clients with diabetes mellitus are older than age 60 and 85% to 90% have type 2. At least 80% of clients diagnosed with type 2 diabetes mellitus are obese.

Which of the following insulins are used for basal dosage? A. Lispro (Humalog) B. Aspart (Novolog) C. NPH (Humulin N) D. Glarginet (Lantus)

Glarginet (Lantus) Lantus is used for basal dosage. NPH is an intermediate acting insulin, usually taken after food. Humalog and Novolog are rapid-acting insulins.

A nurse is teaching a client with type 1 diabetes how to treat adverse reactions to insulin. To reverse hypoglycemia, the client ideally should ingest an oral carbohydrate. However, this treatment isn't always possible or safe. Therefore, the nurse should advise the client to keep which alternate treatment on hand? a) Glucagon b) Hydrocortisone c) 50% dextrose d) Epinephrine

Glucagon During a hypoglycemic reaction, a layperson may administer glucagon, an antihypoglycemic agent, to raise the blood glucose level quickly in a client who can't ingest an oral carbohydrate. Epinephrine isn't a treatment for hypoglycemia. Although 50% dextrose is used to treat hypoglycemia, it must be administered I.V. by a skilled health care professional. Hydrocortisone takes a relatively long time to raise the blood glucose level and therefore isn't effective in reversing hypoglycemia.

During a follow-up visit 3 months after a new diagnosis of type 2 diabetes, a client reports exercising and following a reduced-calorie diet. Assessment reveals that the client has only lost 1 pound and did not bring the glucose-monitoring record. Which value should the nurse measure? A. Glycosylated hemoglobin level B. Fasting blood glucose level C. Glucose via a urine dipstick test D. Glucose via an oral glucose tolerance test

Glycosylated hemoglobin level Glycosylated hemoglobin is a blood test that reflects the average blood glucose concentration over a period of approximately 2 to 3 months. When blood glucose is elevated, glucose molecules attach to hemoglobin in red blood cells. The longer the amount of glucose in the blood remains above normal, the more glucose binds to hemoglobin and the higher the glycosylated hemoglobin level becomes.

When the nurse inspects the feet of a diabetic, a tack is found sticking in the sole of one foot. The client denies feeling anything unusual in the foot. Which is the best rationale for this finding? In diabetes, the autonomic nerves are affected. Motor neuropathy causes muscles to weaken and atrophy. High blood sugar decreases blood circulation to nerves. Nephropathy is a common complication of diabetes mellitus.

High blood sugar decreases blood circulation to nerves. Diabetic neuropathy results from poor glucose control and decreased blood circulation to nerve tissues. The lack of sensitivity increases the potential for soft tissue injury without awareness. Autonomic neuropathy is a complication of diabetes mellitus but not significant with peripheral injuries. Motor neuropathy does occur with poor glucose control but not specific to this injury. Nephropathy is a common complication that directly affects the kidneys.

A client with diabetes mellitus is receiving an oral antidiabetic agent. The nurse observes for which symptom when caring for this client? a. Polyuria b. Hypoglycemia c. Blurred vision d. Polydipsia

Hypoglycemia The nurse should observe the client receiving an oral antidiabetic agent for signs of hypoglycemia. The time when the reaction might occur is not predictable and could be from 30 to 60 minutes to several hours after the drug is ingested. Polyuria, polydipsia, and blurred vision are symptoms of diabetes mellitus.

The nurse is educating the diabetic client on setting up a sick plan to manage blood glucose control during times of minor illness such as influenza. Which is the most important teaching item to include? a) Decrease food intake until nausea passes. b) Increase frequency of glucose self-monitoring. c) Do not take insulin if not eating. d) Take half the usual dose of insulin until symptoms resolve.

Increase frequency of glucose self-monitoring.

A 60-year-old client comes to the ED reporting weakness, vision problems, increased thirst, increased urination, and frequent infections that do not seem to heal easily. The physician suspects that the client has diabetes. Which classic symptom should the nurse watch for to confirm the diagnosis of diabetes? a) Numbness b) Increased hunger c) Fatigue d) Dizziness

Increased hunger The classic symptoms of diabetes are the three Ps: polyuria (increased urination), polydipsia (increased thirst), and polyphagia (increased hunger). Some of the other symptoms include tingling, numbness, and loss of sensation in the extremities and fatigue.

A nurse is providing education to a client who is newly diagnosed with diabetes mellitus. What are classic symptoms associated with diabetes? a) Increased thirst, hunger, and urination b) Loss of appetite, increased urination, and dehydration c) Increased weight loss, dehydration, and fatigue d) Increased weight gain, appetite, and thirst

Increased thirst, hunger, and urination

A nurse is assessing a client who is receiving total parenteral nutrition (TPN). Which finding suggests that the client has developed hyperglycemia? A. Cheyne-Stokes respirations B. Diaphoresis C. Decreased appetite D. Increased urine output

Increased urine output Glucose supplies most of the calories in TPN; if the glucose infusion rate exceeds the client's rate of glucose metabolism, hyperglycemia arises. When the renal threshold for glucose reabsorption is exceeded, osmotic diuresis occurs, causing an increased urine output. A decreased appetite and diaphoresis suggest hypoglycemia, not hyperglycemia. Cheyne-Stokes respirations are characterized by a period of apnea lasting 10 to 60 seconds, followed by gradually increasing depth and frequency of respirations. Cheyne-Stokes respirations typically occur with cerebral depression or heart failure.

The client who is managing diabetes through diet and insulin control asks the nurse why exercise is important. Which is the best response by the nurse to support adding exercise to the daily routine? A. Decreases need for pancreas to produce more cells B. Increases ability for glucose to get into the cell and lowers blood sugar C. Creates an overall feeling of well-being and lowers risk of depression D. Decreases risk of developing insulin resistance and hyperglycemia

Increases ability for glucose to get into the cell and lowers blood sugar Exercise increases trans membrane glucose transporter levels in the skeletal muscles. This allows the glucose to leave the blood and enter into the cells where it can be used as fuel. Exercise can provide an overall feeling of well-being but is not the primary purpose of including in the daily routine of diabetic clients. Exercise does not stimulate the pancreas to produce more cells. Exercise can promote weight loss and decrease risk of insulin resistance but not the primary reason for adding to daily routine.

The nurse is preparing a presentation for a group of adults at a local community center about diabetes. Which of the following would the nurse include as associated with type 2 diabetes? A. Insufficient insulin production B. Little relation to prediabetes C. Less common than type 1 diabetes D. Onset most common during adolescence

Insufficient insulin production Type 2 diabetes is characterized by insulin resistance or insufficient insulin production. It is more common in aging adults and now accounts for 20% of all newly diagnosed cases. Type 1 diabetes is more likely in childhood and adolescence; although, it can occur at any age. It accounts for approximately 5% to 10% of all diagnosed cases of diabetes. Prediabetes can lead to type 2 diabetes.

A nurse explains to a client that she will administer his first insulin dose in his abdomen. How does absorption at the abdominal site compare with absorption at other sites? a) Insulin is absorbed rapidly regardless of the injection site. b) Insulin is absorbed more rapidly at abdominal injection sites than at other sites. c) Insulin is absorbed unpredictably at all injection sites. d) Insulin is absorbed more slowly at abdominal injection sites than at other sites.

Insulin is absorbed more rapidly at abdominal injection sites than at other sites. Subcutaneous insulin is absorbed most rapidly at abdominal injection sites, more slowly at sites on the arms, and slowest at sites on the anterior thigh. Absorption after injection in the buttocks is less predictable.

Which factor presents the most likely cause for weight gain in a diabetic client who is controlled with insulin? Insulin is an anabolic hormone. Insulin provides more efficient use of glucose. Faulty fat metabolism is shut off. Weight gain is attributed to fluid retention.

Insulin is an anabolic hormone. Insulin is an anabolic hormone that is known to cause weight gain. Insulin does lower blood glucose levels by allowing for active transport of glucose into the cells. Faulty fat and protein metabolism will cease once glucose provides the needed the fuel for energy. The restoration of normal metabolism is not the primary cause for weight gain in a client prescribed insulin. Fluid retention is not indicated in this client.

NPH is an example of which type of insulin? Rapid-acting Short-acting Intermediate-acting Long-acting

Intermediate-acting NPH is an intermediate-acting insulin.

Which statement is correct regarding glargine insulin? a) Its peak action occurs in 2 to 3 hours. b) It is absorbed rapidly. c) It cannot be mixed with any other type of insulin. d) It is given twice daily.

It cannot be mixed with any other type of insulin.

The nurse is describing the action of insulin in the body to a client newly diagnosed with type 1 diabetes. Which of the following would the nurse explain as being the primary action? A. It stimulates the pancreatic beta cells. B. It decreases the intestinal absorption of glucose. C. It aids in the process of gluconeogenesis. D. It carries glucose into body cells.

It carries glucose into body cells Insulin carries glucose into body cells as their preferred source of energy. Besides, it promotes the liver's storage of glucose as glycogen and inhibits the breakdown of glycogen back into glucose. Insulin does not aid in gluconeogenesis but inhibits the breakdown of glycogen back into glucose. Insulin does not have an effect on the intestinal absorption of glucose.

Which is a by-product of fat breakdown in the absence of insulin and accumulates in the blood and urine? a) Hemoglobin b) Ketones c) Creatinine d) Cholesterol

Ketones

A patient with a diagnosis of type 2 diabetes has been vigilant about glycemic control since being diagnosed and has committed to increasing her knowledge about the disease. To reduce her risk of developing diabetic nephropathy in the future, this patient should combine glycemic control with what other preventative measure? a) Maintenance of a low-sodium, low-protein diet b) Vigorous physical activity at least three times weekly c) Maintenance of healthy blood pressure and prompt treatment of hypertension d) Subcutaneous injection of 5,000 units of heparin twice daily

Maintenance of healthy blood pressure and prompt treatment of hypertension

A client with diabetes is receiving an oral antidiabetic agent that acts to help the tissues use available insulin more efficiently. Which of the following agents would the nurse expect to administer? Metformin Glyburide Repaglinide Glipizide

Metformin Metformin is a biguanide and along with the thiazolidinediones (rosiglitazone and pioglitazone) are categorized as insulin sensitizers; they help tissues use available insulin more efficiently. Glyburide and glipizide which are sulfonylureas, and repaglinide, a meglitinide, are described as being insulin releasers because they stimulate the pancreas to secrete more insulin.

A client with diabetes is receiving an oral anti diabetic agent that acts to help the tissues use available insulin more efficiently. Which of the following agents would the nurse expect to administer? A. Glipizide B. Glyburide C. Repaglinide D. Metformin

Metformin Metformin is a biguanide and, along with the thiazolidinediones (rosiglitazone and pioglitazone), are categorized as insulin sensitizers; they help tissues use available insulin more efficiently. Glyburide and glipizide, which are sulfonylureas, and repaglinide, a meglitinide, are described as being insulin releasers because they stimulate the pancreas to secrete more insulin.

The nurse is preparing to administer intermediate-acting insulin to a patient with diabetes. Which insulin will the nurse administer? a) Lispro (Humalog) b) Glargine (Lantus) c) NPH d) Iletin II

NPH Intermediate-acting insulins are called NPH insulin (neutral protamine Hagedorn) or Lente insulin. Lispro (Humalog) is rapid acting, Iletin II is short acting, and glargine (Lantus) is very long acting.

A patient who is diagnosed with type 1 diabetes would be expected to: A. Be restricted to an American Diabetic Association diet. B. Have no damage to the islet cells of the pancreas. C. Receive daily doses of a hypoglycemic agent. D. Need exogenous insulin.

Need exogenous insulin Type 1 diabetes is characterized by the destruction of pancreatic beta cells that require exogenous insulin.

A nurse expects to find which signs and symptoms in a client experiencing hypoglycemia? A. Polyphagia and flushed, dry skin B. Polyuria, headache, and fatigue C. Polydipsia, pallor, and irritability D. Nervousness, diaphoresis, and confusion

Nervousness, diaphoresis, and confusion Signs and symptoms associated with hypoglycemia include nervousness, diaphoresis, weakness, light-headedness, confusion, paresthesia, irritability, headache, hunger, tachycardia, and changes in speech, hearing, or vision. If untreated, signs and symptoms may progress to unconsciousness, seizures, coma, and death. Polydipsia, polyuria, and polyphagia are symptoms associated with hyperglycemia.

A characteristic of type 2 diabetes includes which of the following? A. Little insulin B. No islet cell antibodies C. Ketosis-prone when insulin absent D. Often have islet antibodies

No Islet cell antibodies Type 2 diabetes is characterized by no islet cell antibodies or a decrease in endogenous insulin or increase with insulin resistance. Type 1 diabetes is characterized by production of little or no insulin; the patient is ketosis-prone when insulin is absent and often has islet cell antibodies.

A nurse obtains a fingerstick glucose level of 45 mg/dl on a client newly diagnosed with diabetes mellitus. The client is alert and oriented, and the client's skin is warm and dry. How should the nurse intervene? A. Obtain a serum glucose level. B. Obtain a repeat fingerstick glucose level. C. Notify the physician. D. Give the client 4 oz of milk and a graham cracker with peanut butter.

Obtain a repeat fingerstick glucose level The nurse should recheck the fingerstick glucose level to verify the original result because the client isn't exhibiting signs of hypoglycemia. The nurse should give the client milk and a graham cracker with peanut butter or a glass of orange juice after confirming the low glucose level. It isn't necessary to notify the physician or to obtain a serum glucose level at this time.

A client with type 2 diabetes asks the nurse why he can't have a pancreatic transplant. Which of the following would the nurse include as a possible reason? A. Need for exocrine enzymatic drainage B. Need for lifelong immunosuppressive therapy C. Underlying problem of insulin resistance D. Increased risk for urologic complications

Underlying problem of insulin resistance Clients with type 2 diabetes are not offered the option of a pancreas transplant because their problem is insulin resistance, which does not improve with a transplant. Urologic complications or the need for exocrine enzymatic drainage are not reasons for not offering pancreas transplant to clients with type 2 diabetes. Any transplant requires lifelong immunosuppressive drug therapy and is not the factor.

Which intervention is essential when performing dressing changes on a client with a diabetic foot ulcer? A. Applying a heating pad B. Debriding the wound three times per day C. Using sterile technique during the dressing change D. Cleaning the wound with a povidone-iodine solution

Using sterile technique during the dressing change The nurse should perform the dressing changes using sterile technique to prevent infection. Applying heat should be avoided in a client with diabetes mellitus because of the risk of injury. Cleaning the wound with povidone-iodine solution and debriding the wound with each dressing change prevents the development of granulation tissue, which is essential in the wound healing process.

The nurse is teaching a client about self-administration of insulin and about mixing regular and neutral protamine Hagedorn (NPH) insulin. Which information is important to include in the teaching plan? a) There is no need to inject air into the bottle of insulin before withdrawing the insulin. b) When mixing insulin, the regular insulin is drawn up into the syringe first. c) If two different types of insulin are ordered, they need to be given in separate injections. d) When mixing insulin, the NPH insulin is drawn up into the syringe first.

When mixing insulin, the regular insulin is drawn up into the syringe first. When rapid-acting or short-acting insulins are to be given simultaneously with longer-acting insulins, they are usually mixed together in the same syringe; the longer-acting insulins must be mixed thoroughly before being drawn into the syringe. The American Diabetic Association recommends that the regular insulin be drawn up first. The most important issues are that patients (1) are consistent in technique, so the wrong dose is not drawn in error or the wrong type of insulin, and (2) do not inject one type of insulin into the bottle containing a different type of insulin. Injecting cloudy insulin into a vial of clear insulin contaminates the entire vial of clear insulin and alters its action.

A patient is diagnosed with type 1 diabetes. What clinical characteristics does the nurse expect to see in this patient? (Select all that apply.) a) Little endogenous insulin b) Ketosis-prone c) Older than 65 years of age d) Younger than 30 years of age e) Obesity at diagnoses

a) Little endogenous insulin b) Ketosis-prone d) Younger than 30 years of age Type I diabetes mellitus is associated with the following characteristics: onset any age, but usually young (<30 y); usually thin at diagnosis, recent weight loss; etiology includes genetic, immunologic, and environmental factors (e.g., virus); often have islet cell antibodies; often have antibodies to insulin even before insulin treatment; little or no endogenous insulin; need exogenous insulin to preserve life; and ketosis prone when insulin absent.

A nurse is assigned to care for a patient who is suspected of having type 2 diabetes. Select all the clinical manifestations that the nurse knows could be consistent with this diagnosis. a) Wounds that heal slowly or respond poorly to treatment b) Blurred or deteriorating vision c) Sudden weight loss and anorexia d) Fatigue and irritability e) Polyuria and polydipsia

a) Wounds that heal slowly or respond poorly to treatment b) Blurred or deteriorating vision d) Fatigue and irritability e) Polyuria and polydipsia

During a class on exercise for clients with diabetes mellitus, a client asks the nurse educator how often to exercise. To meet the goals of planned exercise, the nurse educator should advise the client to exercise: A. at least three times per week. B. at least five times per week. C. every day. D. at least once per week.

at least three times per week Clients with diabetes must exercise at least three times per week to meet the goals of planned exercise — lowering the blood glucose level, reducing or maintaining the proper weight, increasing the serum high-density lipoprotein level, decreasing serum triglyceride levels, reducing blood pressure, and minimizing stress. Exercising once per week wouldn't achieve these goals. Exercising more than three times per week, although beneficial, would exceed the minimum requirement.

A client has been recently diagnosed with type 2 diabetes, and reports continued weight loss despite increased hunger and food consumption. This condition is called: A. anorexia. B. polyuria. C. polyphagia. D. polydipsia.

polyphagia While the needed glucose is being wasted, the body's requirement for fuel continues. The person with diabetes feels hungry and eats more (polyphagia). Despite eating more, he or she loses weight as the body uses fat and protein to substitute for glucose.

A nurse is providing dietary instructions to a client with hypoglycemia. To control hypoglycemic episodes, the nurse should recommend: a) increasing intake of vitamins B and D and taking iron supplements. b) consuming a low-carbohydrate, high-protein diet and avoiding fasting. c) increasing saturated fat intake and fasting in the afternoon. d) eating a candy bar if light-headedness occurs.

consuming a low-carbohydrate, high-protein diet and avoiding fasting. To control hypoglycemic episodes, the nurse should instruct the client to consume a low-carbohydrate, high-protein diet, avoid fasting, and avoid simple sugars. Increasing saturated fat intake and increasing vitamin supplementation wouldn't help control hypoglycemia.

A client tells the nurse that she has been working hard for the past 3 months to control her type 2 diabetes with diet and exercise. To determine the effectiveness of the client's efforts, the nurse should check: a) glycosylated hemoglobin level. b) fasting blood glucose level. c) serum fructosamine level. d)urine glucose level.

glycosylated hemoglobin level. Because some of the glucose in the bloodstream attaches to some of the hemoglobin and stays attached during the 120-day life span of red blood cells, glycosylated hemoglobin levels provide information about blood glucose levels during the previous 3 months. Fasting blood glucose and urine glucose levels give information only about glucose levels at the point in time when they were obtained. Serum fructosamine levels provide information about blood glucose control over the past 2 to 3 weeks.

Which combination of adverse effects should a nurse monitor for when administering I.V. insulin to a client with diabetic ketoacidosis? A. Hypocalcemia and hyperkalemia B. Hyperkalemia and hyperglycemia C. Hypokalemia and hypoglycemia D. Hypernatremia and hypercalcemia

hypokalemia and hypoglycemia Blood glucose needs to be monitored in clients receiving I.V. insulin because of the risk of hyperglycemia or hypoglycemia. Hypoglycemia might occur if too much insulin is administered. Hypokalemia, not hyperkalemia, might occur because I.V. insulin forces potassium into cells, thereby lowering the plasma level of potassium. Calcium and sodium levels aren't affected by I.V. insulin administration.

A nurse is preparing the daily care plan for a client with newly diagnosed diabetes mellitus. The priority nursing concern for this client should be: A. administering insulin routinely and as needed via a sliding scale. B. checking for the presence of ketones with each void. C. providing client education at every opportunity. D. monitoring blood glucose every 4 hours and as needed.

providing client education at every opportunity The nurse should use routine care responsibilities as teaching opportunities with the intention of preparing the client to understand and eventually manage his disease. Monitoring blood glucose, checking for the presence of ketones, and administering insulin are important when caring for a client with diabetes, but they aren't the priority of care.

A nurse is assigned to care for a postoperative client with diabetes mellitus. During the assessment interview, the client reports that he's impotent and says he's concerned about the effect on his marriage. In planning this client's care, the most appropriate intervention would be to: A. provide support for the spouse or significant other. B. suggest referral to a sex counselor or other appropriate professional. C. encourage the client to ask questions about personal sexuality. D. provide time for privacy.

suggest referral to a sex counselor or other appropriate professional The nurse should refer this client to a sex counselor or other professional. Making appropriate referrals is a valid part of planning the client's care. The nurse doesn't normally provide sex counseling.


Kaugnay na mga set ng pag-aaral

Micronutrients FINAL water and minerals

View Set

Hesi Quiz: Health Policy/Systems - Health Care Law

View Set

meteorology ch15 critical questions and review questions

View Set

Abrams Chapter 9 - Drug Therapy for Coagulation Disorders, Chapter 10: Drug Therapy for Dyslipidemia, Chapter 27 Dysrhythmias, Chapter - 27 Drug therapy for dysthymia, Pharm Chapter 27 Drug Therapy for Dysrhythmias

View Set

General Functions of the Nervous System

View Set